[Ответить в тред] Ответить в тред



<<
Назад | Вниз | Каталог | Обновить тред | Автообновление
613 | 74 | 152

Тред тупых вопросов №85 Аноним 21/02/18 Срд 07:59:58  383656  
Einstein and St[...].jpg (122Кб, 970x599)
black-hole-merg[...].jpg (587Кб, 1200x800)
HYdfv8N.mp4 (1517Кб, 720x570, 00:00:15)
image.png (1177Кб, 1032x774)
Тред вопросов о жизни, Вселенной и всем таком.

Спрашиваем то, за что в других местах выдают путёвку в биореактор. Здесь анонимные ученые мирового уровня критически рассмотрят любые гениальные идеи и нарисованные в Paint схемы.

Предыдущий тут: >>379584 (OP) https://2ch.hk/spc/res/379584.html

Q: Можно быстрее?
A: Можно упасть в пузырь альбукерке, наса уже почти надула его.

Q: Я начитался охуительных историй про уфологию, че делать, нам жопа?
A: Да, тебе жопа, можешь сгонять в зогач или куда оттуда пошлют.

Q: Что будет с человеком в вакууме без скафандра / если он упадет на черную дыру / попробует ступить на поверхность газового гиганта/солнца?
A: Он умрёт.
Аноним 21/02/18 Срд 08:35:09  383666
>>383656 (OP)
>упасть в пузырь альбукерке
Ебанный стыд...
Во-первых, Алькубьерре.
Во-вторых, не упасть, а создавать вокруг корабля изнутри (иначе кина не будет).
В-третьих, НАСА искривляет пространство на десятимиллионную часть, контролируя это сверхточными интерферометрами, до самого варп-привода здесь - как до Антарктиды раком.
Аноним 21/02/18 Срд 08:36:53  383667
>>383666
Кстати, что там с пузырем? Уже сколько лет прошло, а никаких новостей про пузырь и ведро. Неужто физику не наебать и это все просто сенсационализм?
Как там локхидовский термоядерный реактор в фуре, кстати?
Аноним 21/02/18 Срд 09:36:33  383679
Если пельмени с Сатурна отлучить от колец, что станет с их формой?
Аноним 21/02/18 Срд 09:52:37  383680
>>383667
В фуре?
Аноним 21/02/18 Срд 09:55:49  383681
>>383680
Да, обещали впихнуть реактор что его можно грузовиком перевозить.
Аноним 21/02/18 Срд 10:03:05  383684
>>383656 (OP)
>>383666
Канонiчный тред, батя грит малаца.
Аноним 21/02/18 Срд 10:05:34  383686
>>383681
>обещали
обещать и не дать а дурачку радость
Аноним 21/02/18 Срд 10:43:06  383693
>>383656 (OP)
какой страшный трап с Карлом Марксом, да ещё и соски видны
Аноним 21/02/18 Срд 11:28:43  383706
>>383693
этого трапа звали Фридрих Энгельс.
Аноним 21/02/18 Срд 19:45:58  383851
Допустим у нас есть скафандр, который помогает человеку справиться с вакуумом, но не обладает никакой теплоизоляцией и никак не удерживает тепло. Человек в таком скафандре попадает в открытый космос в тень от планеты. Через какое время он потеряет все тепло и умрет от холода? Именно от холода? Он же не начнет мгновенно умирать, как будто он попал в холодильник?
Аноним 21/02/18 Срд 20:03:04  383863
image.png (472Кб, 729x887)
>>383693
>>383706
>2022
>не знать знаменитого гомосексуалиста Стивена Фрая и его мужа.
Аноним 21/02/18 Срд 20:04:34  383866
>>383851
Нет. Он умрет от теплового шока.
Потому, что тело в вакууме не сможет рассеивать более 100Вт внутреннего тепла все равно.
Это умерший человек с остановившимся метаболизмом и погибшими бактериями ответственными за разложение будет остывать. Живой перегреется.
Аноним 21/02/18 Срд 20:11:23  383869
93c6f97b336febd[...].png (1054Кб, 1103x574)
>>383667
>Как там локхидовский термоядерный реактор в фуре, кстати?
Описание концепции: Открытая ловушка с внутриплазменными диполями. Позволяет, как и все открытые ловушки достичь высокой доли использования давления магнитного поля (в отличии от токамаков), а значит доступны реакции DD, DHe3.

Экспертная позиция: На самом деле, этот тип магнитных ловушек концептуально уходит еще в 60е годы, и плотно исследовался теоретически и экспериментально. Но похоже, команда CFR истории не знает, поэтому набивает многие шишки самостоятельно. Интересно, что изначальные планы создать реактор “который помещается в грузовик” уже отброшены, и размеры минимального реактора подросли до 7х22 метра. Если дальше экстраполировать опыт работы с этой концепцией на команду CFR, то скоро они должны узнать, что размещать сверхпроводящие катушки прямо внутри идущей термоядерной реакции мягко говоря “не инженерно” и из лаборатории этой концепции скорее всего не удасться выбраться и в этот раз.
https://geektimes.ru/post/297461/
Аноним 21/02/18 Срд 20:12:52  383870
>>383851
Он же не начнет мгновенно умирать, как будто он попал в холодильник?
Нет. Поскольку человек в скафандре ни с чем не соприкасается и свои пуки наружу не выпускает, то и теплопередачи почти нету. Вся потеря тепла идет только через излучение, а это довольно медленно. С другой стороны, космонавты при работе в открытом космосе за 45 минут в тени успевают замерзнуть даже с работающей системой климат-контроля. Думаю, человек без дополнительного обогрева больше 12 часов не проживет. Но вообще должен раньше свариться.
Аноним 21/02/18 Срд 20:38:49  383883
>>383851
Я тут мимопроходил, но как-то видел похожий вопрос где-то на кворе. Там отвечали, что голый (но живой за счёт волшебной силы мысленного эксперимента) мужик в космосе остынет с нормальных 36 до "пиздец гарантирован" 30 градусов примерно за полчаса.
Аноним 21/02/18 Срд 20:58:39  383888
>>383866
Этот анон говорит, что от теплового шока человек умрет, но в целом от перегрева.

>>383870
Этот анон говорит, что человек с учетом охлаждения скафандра может умереть через 45 минут, но одновременно он должен раньше свариться. В тени!

>>383883
Этот анон говорит, что человек остынет за полчаса до 30 градусов и пиздец.

Кому верить?
Аноним 21/02/18 Срд 20:58:42  383889
>>383851
Туловище человека возьмем за цилиндр общей площадью в 2 квадратных метра, температуру тела за 310 кельвинов, мощность внутреннего источника тепла за 100 ватт.
Если мощность излучения больше его внутреннего источника - он замерзнет (или достигнет теплового равновесия со своим источником), если больше - то сварится (или достигнет, опять же).
Мощность это сигма помноженная та температуру в четвертой степени (Стефана-Больцмана используем).
Сигма =5,670367 х 10-8 Вт·м−2·К−4.
Температура в четвертой = 9 235 210 000.
Умножаем и получаем 523 ватта.
Остыв на 10 градусов до нежизнепригодных 26 градусов цельсия он все равно будет излучать 460 ватт тепла.
Ну а скорость охлаждения предоставим посчитать нашим юным зрителям.
Считайте мощность излучения в 400 ватт, массу человека в 70кг и теплоемкость равную теплоемкости воды.
Аноним 21/02/18 Срд 21:01:17  383890
image.png (258Кб, 600x600)
>>383888
Тому, кто считал >>383889
Аноним 21/02/18 Срд 21:21:03  383893
>>383888
Этот анон >>383870 то есть я имел ввиду, что сваришься на Солнце, потому что в тени 12 часов ты не будешь торчать. Но если вдруг представить, что ты спрятался от Солнца, то от переохлаждения. И не за 45 минут, а за много часов. По моему диванному чутью около 12.

>>383889
А этот анон пускай объяснит тот факт, что космонавты мерзнут в тени, если он по-прежнему утверждает, что человеческое тело выделяет энергии больше, чем излучает.
Аноним 21/02/18 Срд 21:29:23  383896
>>383893
В глаза не долбись. Вырабатываешь 100 ватт, излучаешь 500 ватт. Мерзнешь.
Аноним 22/02/18 Чтв 00:11:47  383934
Какая ракета самая толстая относительно общих габаритов? Энергия?
Аноним 22/02/18 Чтв 00:13:04  383937
>>383934
Н-1
Аноним 22/02/18 Чтв 00:36:04  383943
>>383937

Но у нее только низ толстый.
Аноним 22/02/18 Чтв 00:46:53  383950
1200px-ProtonZv[...].jpg (202Кб, 1200x1747)
>>383943
Тогда мамка твоя Протон
Аноним 22/02/18 Чтв 00:55:29  383952
>>383950
Энергия потолще.
Аноним 22/02/18 Чтв 01:00:19  383954
>>383952
Я думал тебе уже надо только летающие сейчас. Про энергию забыл, да.
Аноним 22/02/18 Чтв 02:04:40  383971
image.png (873Кб, 768x780)
>>383954
Как будто Н1 летает.
А вообще Ариан вроде толще, не?
Аноним 22/02/18 Чтв 02:14:10  383973
>>383971
Нет
Аноним 22/02/18 Чтв 12:38:37  384035
Возможно ли существование более-менее выраженных колец у звезды? Пояс астероидов можно считать кольцом?
Аноним 22/02/18 Чтв 12:39:19  384037
>>383656 (OP)
>HYdfv8N.mp4
Это взрывная волна в космосе, да?
Аноним 22/02/18 Чтв 12:42:08  384039
>>384037
Возможно, гравитационные волны показаны как взрывная волна.
Если ты про начальные искажения.
Дальнейшее - это разброс вещества.
Аноним 22/02/18 Чтв 13:03:19  384042
>>383656 (OP)
>HYdfv8N.mp4
Я когда показал эту эмпечетыречу здеся и попросил чекнуть на твёрдость относительно нашего сраного мироздания, меня обосрали :(
Аноним 22/02/18 Чтв 13:05:53  384043
>>384042
Показывай, где обосрали, кто обосрал, пообщаемся с ними.
Нетвердая, конечно, но и не слишком мягкая.
Аноним 22/02/18 Чтв 13:10:58  384044
>>384043
Сказали что я купился на пафосную надпись "Эта лучшая визуализация Сверхновой" И что ученые десятилетиями мудохаются составляя модели взрыва сверхновых и распространения говн звезды в процессе взрыва. А тут типа дизайнер задизайнил и усё. И ещё УДАРНЫЕ ВОЛНЫ В КОСМАЦЕ!!1!!! Я тоже сказал что это гравитационные, но они типа - они не могут так явно искажать просранство, это же не черная дыра, правда про черную они не говорили..

Ещё мемас написали типа:
Этот комментарий признан лучшим в треде
Аноним 22/02/18 Чтв 13:20:53  384045
>>384044
Не переживай. Это и правда очень красивая визуализация. Сделанная для того, в первую очередь, чтобы быть красивой.
Аноним 22/02/18 Чтв 14:38:06  384057
>>384035
>более-менее выраженных колец у звезды?
Только протопланетарный диск если. Но он больше тороидальный, в отличие от планетарных колец.
>Пояс астероидов можно считать кольцом?
Можно считать, а можно не считать. В астероидном поясе довольно крупное говно летает, в отличии от.
Аноним 22/02/18 Чтв 15:51:39  384061
>>383889
>мощность внутреннего источника тепла за 100 ватт

откуда?
Аноним 22/02/18 Чтв 15:53:16  384062
>>384061
Из метаболизма же.
Или ты про то, что не упомянут аппарат дыхания, человек останавливается и перестает кровоток и метаболизм тоже?
Аноним 22/02/18 Чтв 15:54:47  384063
>>383851
>сть скафандр, который помогает человеку справиться с вакуумом, но не обладает никакой теплоизоляцией и никак не удерживает тепло. Человек в таком скафандре попадает в открытый космос в тень от планеты. Через какое время он потеряет все тепло и умрет от холода? Именно от холода? Он же не начнет мгновенно умирать, как будто он попал в холодильник?


Человек перегреется. Умные люди все посчитали и потому оснащают скафандры испарителями водички для охлаждения.
Аноним 22/02/18 Чтв 15:57:52  384065
>>384062
Само значение откуда?
Аноним 22/02/18 Чтв 16:07:15  384067
>>384065
Из биологии. Метаболизм это процессы работы организма когда в клектах окисляется и горит еда и микроэлеме
Аноним 22/02/18 Чтв 16:51:48  384078
>>384067
>еда и микроэлеме
Хорошо, что тебя подстрелил дежурный снайпер, потому что тот хуй спрашивал не про значение слова метаболизм, а откуда взято значение в 100 ватт. Если автор поста >>384062 тоже ты, то я даже не знаю, тралируешь ты или просто не очень умный.
Аноним 22/02/18 Чтв 18:21:36  384128
Вот говорят в космосе без скафандра не замёрзнешь. Но ведь там влага с поверхности испаряется почти мгновенно, не должен ли человек как минимум окаченеть?
Аноним 22/02/18 Чтв 19:14:53  384166
245015314110573.jpg (63Кб, 755x973)
245111235110119.jpg (9Кб, 347x233)
Нахуя нужно это йоба охлаждение?
Аноним 22/02/18 Чтв 19:17:21  384167
>>384166
Это система шумоподавления.
Аноним 22/02/18 Чтв 19:17:29  384168
>>384166
Это не охлаждение.
Это вибро- и шумопоглощение.
Аноним 22/02/18 Чтв 19:18:57  384170
>>384166
Подавление виброакустических нагрузок на ракету и стенд и защита оборудования площадки от лишнего перегрева.
Аноним 22/02/18 Чтв 19:20:04  384172
>>384168
А нахуя она нужна?
Аноним 22/02/18 Чтв 19:21:02  384174
>>384172
Чтобы было меньше шума и вибраций.
Аноним 22/02/18 Чтв 20:06:46  384182
>>384061
>откуда?
Отовсюду.
http://hyperphysics.phy-astr.gsu.edu/hbase/thermo/bodrad.html

http://ergo.human.cornell.edu/studentdownloads/DEA3500notes/Thermal/thcondnotes.html
Аноним 22/02/18 Чтв 20:16:16  384183
1. Возможно ли, что прямо сейчас на далеких планетых живут крупные животные размером с льва или слона и миллиона лет даже не развиваются? Если да, то представьте, как там лампово, только дикая природа и ничего более.
2. Возможна ли эволюция в океане, как Левиафаны из масс эффекта, может ли здоровый зверь из океана стать разумным?
3. Если на Земле внезапно появится другой разумный вид, к примеру обезьяны внезапно обретут разум и научаться говорить, то что будут делать люди? Обращаться с ними как с равными? Переселять в города? Или что?
4. Если все люди внезапно исчезнут, то возможно ли, что кто-то из нынешних животных через миллионы лет даст путь развития другому разумному виду? Если да, то кто?
5. Правда ли, что дельфины такие умные, что могут мыслить? Мне кажется это пиздеж.
6. Правда, что на Земле будет постапокалипсис как в мэд максе на начальных этапах сумирания солнца и будут охуенные атмосферные пейзажи как в фильмах, только ирл?
Аноним 22/02/18 Чтв 20:28:31  384193
>>384183
1. Да.
2. Да. Вряд ли - без рук и огня же.
3. Внезапно не появится, миллионы лет надо. Расистская шуточка про негров.тхт
4. Возможно. Приматы, медведи, кенгуру, птицы типа врановых или попугаевых.
5. Могут общаться и складывать предложения, имеют свою речь. Из-за того, что они акватики их разум сложно характеризовать и структурировать как разум шимпанзе или горилл, которые на уровне 3-летнего ребенка.
6. Нет. Будет буквальный апокалипсис и все умрут. Охуенные пейзажи есть и сейчас.
Аноним 22/02/18 Чтв 20:39:19  384199

>>384183
https://www.youtube.com/results?search_query=Станислав+Дробышевский
Аноним 22/02/18 Чтв 21:01:30  384205
e001b0568d8152c[...].jpg (113Кб, 660x989)
Ребята, почему нельзя убрать первую ступень и не поднять ракету на каком-нибудь аэростате?
Аноним 22/02/18 Чтв 21:05:24  384206
>>384205
Спасибо, надо не забыть в следующую шапку добавить.
Уже не первый тред подряд задают эту платину.
>>375335
>Почему не поднимают ракеты до стратосферы на воздушном шаре?
Читани ответы.
Аноним 22/02/18 Чтв 21:12:05  384210
>>384205
Да вы сговорились сука что ли? Каждый тред один и тот же вопрос.
Аноним 22/02/18 Чтв 21:16:00  384212
>>384206
>в следующую шапку добавить.
Предлагаю так:
Q: Почему не поднимают ракеты до стратосферы на воздушном шаре?
A: ААА БЛЯДЬ ИДИ НАХУЙ
Но я не настаиваю, всё равно шапку никто не будет читать.
Аноним 22/02/18 Чтв 21:16:47  384213
>>384210
Ты шутишь что ли? Это самый платиновый вопрос популярнее, чем "можно ли превысить скорость света".
Просто умножь на мертвоту доски. Если б доска была живуча как хотя бы веге, то тут бы чаще чем раз в день задавали этот вопрос.
Бля, вот бы увидеть спейсач таким оживленным хотя б денек-другой...

>>384212
Как вариант. Но я бы просто вложил ссылку на чтоес.ли где про это подробно написано.
Аноним 22/02/18 Чтв 21:28:13  384217
>>384213
>Бля, вот бы увидеть спейсач таким оживленным хотя б денек-другой...
Да ну нахуй. Как по мне, сейчас живость пейсача в самый раз - и не глухо, и за всем можно уследить. Плохо только что общее "качество" постинга на борде упало параллельно с повышением её посещаемости, ну это так всегда и везде наверное. Ты видать мертвоту доски не застал, когда бывало и по 5 постов в день на ВСЮ борду (ну если не считать огурцетред конечно).
Аноним 22/02/18 Чтв 21:31:51  384220
>>384193
>6.
Я не смотрел мед Макса, но разве солнце не будет нагреватся постепенно на протяжении миллионов лет?
Аноним 22/02/18 Чтв 21:34:00  384222
>>384220
Да. И это приведет к нежизнеспособности планеты. К буквальному апокалипсису. Постапокалипсис по определению наступает после апокалипсиса. Постапокалипсиса от раздувания солнца не случится. Случится апокалипсис на чем все и закончится для этой планеты.
Аноним 22/02/18 Чтв 21:41:50  384224
>>384222
Вот чё ты выёбываешься? Ты же прекрасно понял в чём суть вопроса, нет надо доебаться.
Аноним 22/02/18 Чтв 22:05:57  384232
>>384217
>5 постов в день на ВСЮ борду (ну если не считать огурцетред конечно).
Бывало и 5 постов в день включая посты огурцача. Летом в выходные, например.
Аноним 22/02/18 Чтв 22:07:53  384234
>>384217
Помню я эти времена. Перекатывал некоторые огуречные треды даже.

С огурцами поживее было как-то. Ньюфаги чаще забегали.
А щас как-то все расселись по тредам где занимаются цирклджеркингом и иногда забегают в соседний тред потралировать.
Аноним 22/02/18 Чтв 22:15:27  384238
>>384205
Вот пара хороших ответов на этот вопрос
>>375337
>>362256
Аноним 22/02/18 Чтв 22:44:55  384244
>>384224
>нет надо доебаться.
Ты в спейсаче. Тут если не доебаться, то ракета с гептилом на голову упадёт.
Аноним 22/02/18 Чтв 23:07:18  384249
Есть ли более-менее точные цифры по умершим от космонавтики? Не прямые потери типа угаревших шаттлов, а побочные, всякие взрывы на испытаниях, несчастные случаи, смерть от гептила и прочие происшествия.
Аноним 22/02/18 Чтв 23:15:32  384250
>>384244
Мы же не в Китае.
Аноним 22/02/18 Чтв 23:16:28  384251
>>384249
Есть. Иди по ссылке и мотай до раздела Non-astronaut fatalities:
https://www.wikiwand.com/en/List_of_spaceflight-related_accidents_and_incidents
Узнаешь много нового, например как сегмент твердотопливного ускорителя от Титанауронили на пол и тот запустился нахуй, как мужики на падали с обслуживающих башен, придавливались кран-балками и лишались головы из-за потока воды из труб высокого давления. Правда там не очень подробно.
Аноним 22/02/18 Чтв 23:17:54  384252
>>384249
> смерть от гептила
Вот тут уже хуй посчитаешь.
Так придумать, то можно вообще вплоть до упавших на стройке завода по сжижению кислорода считать.
Взрывы засекречены некоторые, так что хрен посмотришь. Тот же интелсат 708 упал на город, так одни говорят, что две тысячи фрагов, другие, что сотня-другая, а официальные власти что город вообще был выселен и никто не пострадал.
Аноним 22/02/18 Чтв 23:40:51  384253
screenshot-www.[...].png (49Кб, 788x760)
screenshot-www.[...].png (13Кб, 779x195)
>>384061
>>384065
>>384078
>откуда взято значение в 100 ватт
Из базового курса физиологии человека.
https://www.wolframalpha.com/input/?i=metabolic+rate+30+yr+70+kg+174+cm+male
Вот значения для сферическогосреднего космонавта в вакууме, при полной неподвижности и при легкой работе
https://www.wolframalpha.com/input/?i=1671+Cal%2Fday+in+watts;+2298+Cal%2Fday+in+watts
Аноним 23/02/18 Птн 14:30:30  384313
Проводили ли космонавты научные опыты по дракам в невесомости?
Аноним 23/02/18 Птн 14:42:17  384314
1519386101916-1[...].jpg (217Кб, 333x500)
>>384313
Аноним 23/02/18 Птн 14:43:06  384316
fb15d5c7cffa5d4[...].jpg (44Кб, 451x392)
>>384313
Проиграл.
Какая ценность таких исследований?
Может быть астронавты на вомит комете тренировались отбиваться и целиться из пистолей, разве что так. ИРЛ никакого практического применения такому нет.
Аноним 23/02/18 Птн 19:03:35  384324
Сильный холод на Нептуне-Уране или Титане будет влиять на нагрев при вхождении? Опасность атмосферного угара снижается при прочих равных?
Аноним 23/02/18 Птн 19:19:51  384325
>>384324
Да вообще поебать, нагрев же идёт из-за аэродинамического сопротивления, какая бы температура не была перед ударной волной, за ней она уже будет ультраебическая.
Ну газ может быть плотнее или разреженнее в зависимости от температуры, так что из-за холода вход наверное будет даже жёстче на полшишечки.
Аноним 23/02/18 Птн 19:47:12  384326
Как должна выглядеть центрифужная катапульта для запуска в космос?

https://twitter.com/BryceSpaceTech/status/967072985571057665
Аноним 23/02/18 Птн 20:31:43  384334
>>383666
>НАСА искривляет пространство на десятимиллионную часть
То есть мы уже искривляем пространство намеренно?
Аноним 23/02/18 Птн 20:37:46  384335
>>384316
> ИРЛ никакого практического применения такому нет.
А захватить МКС?
Аноним 23/02/18 Птн 20:54:08  384337
>>384335
Захватывай. Серьезно.
Аноним 23/02/18 Птн 20:58:40  384339
>>384337
Круто же будет, пиндосов в шлюз и мкснаш.
Аноним 23/02/18 Птн 20:59:35  384340
>>384339
Дерзай, тебе никто не мешает.
Аноним 23/02/18 Птн 21:02:34  384342
>>384340
цингусы, драконы и законы с соглашениями
Аноним 23/02/18 Птн 22:49:55  384356
image.png (1424Кб, 2000x2000)
image.png (246Кб, 1600x600)
Почему у телескопов такие простые названия? Это какая-то традиция?
Аноним 23/02/18 Птн 22:54:12  384358
image.png (1442Кб, 2000x2000)
>>384356
А что сняли на эти телескопы?
Аноним 23/02/18 Птн 23:24:07  384363
1519012587656.jpg (1163Кб, 1638x3053)
>>384358
в court судят лалка
Аноним 23/02/18 Птн 23:31:16  384365
>>384205
>почему не на орлах?
Аноним 23/02/18 Птн 23:36:15  384368
>>383656 (OP)
Были ли прожекты бомбардировки небесных тел примитивными организмами?
Аноним 23/02/18 Птн 23:37:02  384369
>>384368
Поговаривают про терраформинг венеры. Но всерьез никто это не обсуждает. Может кто кандидатские на эту тему защищает и все.
Аноним 23/02/18 Птн 23:37:33  384371
>>384368
каких еще тел блядь? в нашей системе кроме земли ничего для жизни не пригодно
Аноним 23/02/18 Птн 23:46:37  384375
>>384371
Почему?
Аноним 23/02/18 Птн 23:49:36  384378
>>384375
Так боженька повелел.

>>384371
Для классической - да. Педоф Экстремофилы могут всякое, некоторые ажно на наружной стороне МКС поселились и плодятся, например.
Аноним 23/02/18 Птн 23:52:55  384379
>>384371
Про Энцелад забыл.
Аноним 23/02/18 Птн 23:57:53  384381
>>384378
>Так боженька повелел
Что он повелел? Жизнь это просто цепочка химических реакций, для неё необходим только запас сырья и энергия. Как эти реакции на земле протекают, так и на венере будут протекать. Почему там не может появиться молекула белка?
Аноним 23/02/18 Птн 23:59:27  384382
>>384381
Потому, что там 500 градусов цельсия и сто атмосфер углекислого газа.
Аноним 23/02/18 Птн 23:59:37  384383
>>384381
Потому что там температуры выше температуры денатурации белка, например.
Аноним 24/02/18 Суб 00:21:57  384389
1519420846989-6[...].jpg (1788Кб, 1536x1371)
Это Фотошоп или там на самом деле так? Зачем?
Аноним 24/02/18 Суб 00:44:13  384391
>>384389
Не фотошоп. Спирит там просто разворачивался.
http://www.huffingtonpost.co.uk/2013/04/24/mars-rover-penis-nasa_n_3144656.html
Аноним 24/02/18 Суб 02:38:19  384401
>>383656 (OP)
Впервые у вас на доске и сразу в первых попавшихся тредах вижу что-то про эмбриона, роды и невесомость.

Поясните, что с этим не так?
Это платиновый вопрос, да?
Аноним 24/02/18 Суб 02:39:50  384402
>>384401
Люди в невесомости не беременели, хуй знает. Но по идее для развития плода нужна гравитация, а то вырастет ебанутым.
Аноним 24/02/18 Суб 03:25:24  384405
Что там с проектом Хокинга по запуску фольги к альфе Центавра?
Вроде даже денег 200 млн собрал
Аноним 24/02/18 Суб 09:05:43  384419
>>384363
Ролл
Аноним 24/02/18 Суб 09:07:18  384420
>>384402
А крысы да. Но там плод вроде не пошёл или эмбрион рано сдох
Аноним 24/02/18 Суб 10:24:13  384424
>>384401
>Впервые у вас на доске и сразу в первых попавшихся тредах вижу что-то про эмбриона, роды и невесомость.

В невесомости увеличивается вероятность, что зачатое яйцо приклеится к фаллопиева трубе. а не к матке. А это - несостоявшаяся беременность.
Аноним 24/02/18 Суб 10:26:12  384425
>>384424
А разве большая часть яиц не оплодтворяется непосредственно в матке?
Я девственник, так что никогда не интересовался и позабыл школьную биологию
Аноним 24/02/18 Суб 10:33:14  384426
>>383656 (OP)
Почему почти все говорят, что жизни за пределами земли нет, если космос бесконечный и в теории таких планет как земля может быть дохуища?
Аноним 24/02/18 Суб 10:37:40  384427
>>384426
Космос конечный. В теории таких планет может быть дохуища.
Но ни одной не нашли.
Вот когда найдут жизнь, тогда и будут говорить.
Пока не нашли и не доказали - нет смысла обсуждать. Все равно, что считать, что где-то там звездные войны происходят, а в той галактике Эквестрия с волшебством и дружбой.
Аноним 24/02/18 Суб 10:38:23  384428
>>384426
https://ru.m.wikipedia.org/wiki/Парадокс_Ферми
Аноним 24/02/18 Суб 19:34:49  384471
>>384426
Это называется парадокс Ферми.
Так-то да, вроде инопланетян должно быть дохуя, но мы их вообще не видим. В этом и парадокс.

Пользуясь случаем, спрошу сам.
Есть ли вообще хоть теоретический шанс увидеть следы деятельности инопланетян в других галактиках?
Это насколько они должны там насрать, чтобы даже из другой галактики видно было?
Это абсолютно анрили?
Аноним 24/02/18 Суб 20:58:42  384478
Часто слышал что Юпитер защищает Землю и другие планеты от астероидов и комет, но как это работает? Ясен хуй что он сильно влияет на их орбиты, но в чем защита, это же просто изменение траектории без какой-либо осмысленности.
Аноним 24/02/18 Суб 21:02:57  384479
PGkpGT0.jpg (2231Кб, 500x500)
>>384478
С осмысленностью. Собирает их орбиты в такие конфигурации.
Аноним 24/02/18 Суб 21:03:31  384480
>>384479
>.jpg
Это анимация, кстати, так что можно открывать.
Аноним 24/02/18 Суб 21:14:57  384481
>>384479
Ммм, т.е. он подминает весь пояс, но что с летающими как попало телами?
Кстати в каком месте Церера на гифке?
Аноним 24/02/18 Суб 21:16:56  384482
742px-CeresOrbi[...].png (96Кб, 742x563)
>>384481
У Цереры своя атмосфера.
Аноним 24/02/18 Суб 22:37:27  384487
>>384381
Плюс еще для всей земной жизни нужна вода в жидком виде, которой на Венере уже миллиард лет как нет. И вообще кроме Земли нигде в Солнечной системе жидкой воды на поверхности нет, так что просто скидывать микроорганизмы на поверхность бессмысленно, они там сразу и сдохнут.

Можно зато успешно насрать микроорганизмами в подповерхностные океаны разных спутников типа Европы или Энцелада, но там шибко глубоко бурить надо.
Аноним 24/02/18 Суб 23:28:29  384496
>>384425
Нет, чаще всего в трубах.
Аноним 25/02/18 Вск 00:36:08  384499
>>384487
Венера пригодна для жизни норм, все где есть атмосфера может быть пригодна. На определенной высоте бактерии могут жить и переноситься ветрами. Вниз они будут спускаться и там уже естественный отбор пойдет.
В некоторых местах Солнечной системы микроорганизмы могут уйти хоть в камни, в трещины, на глубину, в лед и начать там свою экосистему строить, в которой они будут пожирать друг друга и свое говно, постепенно эволюционируя.
Аноним 25/02/18 Вск 00:53:21  384502
>>384499
Только развиться во что-то сложное в таких условиях они не смогут.
Аноним 25/02/18 Вск 00:57:03  384503
image.png (258Кб, 600x600)
>>384502
А кто сказал, что условия будут теми же?
Если что, на момент зарождения жизни в атмосфере земли свободного кислорода не было.
Аноним 25/02/18 Вск 00:58:13  384504
>>384502
да хуй знает. Смотря о каких временных промежутках мы говорим(миллионы, миллиарды), смотря как условия изменятся на планете из-за самих мразей, или из-за светимости своньца, орбитальных характеристик, погоды.
Аноним 25/02/18 Вск 00:59:32  384505
>>384503
Если что, Венера этот этап давно прошла и лучше там уже не будет. У нас через миллиарды лет солнце вообще-то начнет раздуваться.
Про терраформинг не надо, не прощу.
Аноним 25/02/18 Вск 01:01:13  384506
>>384479
Хуле орбита треугольная?
Аноним 25/02/18 Вск 01:01:25  384507
>>384505
Ну тогда да, не смогут. Your point being?
Если что, Земля как раз пример биологического терраформинга, весь свободный кислород биогенный.
Потому если заводить речь о спасении жизни как таковой то почему бы и нет?
Аноним 25/02/18 Вск 01:02:12  384508
>>384063
я вспомнил как в армии бегал в костюме ОЗК в минус 10 и это было как в бане, потом из чулок воду выливал.
Аноним 25/02/18 Вск 01:03:00  384509
KkUepVk.gif (80Кб, 480x480)
>>384506
ИЛЛЮЗИЯ.
Орбиты у всех эллиптические, но из-за совпадения периодов и влияния массы Юпитера они выглядят будто в формации "треугольник"
Аноним 25/02/18 Вск 01:05:26  384510
>>384509
Хмм, действительно. Чудеса какие
Аноним 25/02/18 Вск 01:11:49  384513
>>384505
а марс наоборот станет более живым. Нужно просто насрать там, а потом из нашего говна родится новая жизнь.
Аноним 25/02/18 Вск 01:15:25  384514
>>384507
Если так посмотреть то на Земле стартовые условия были получше чем на Венере сейчас.
>>384513
Если я правильно помню, то у Марса будет не очень много времени пока солнце его не поглотит.
Аноним 25/02/18 Вск 01:19:03  384516
>>384514
Щас неизвестно, где еще лучше были. И пока мы не заспамили хорошими зондами другие планеты, непонятно, где стартовые условия были лучше.
Гадать не будем, но соглашусь - венера щас полнейший ад и бесперспективняк.
Марс хотя бы можно заселить в определенных масштабах.
Аноним 25/02/18 Вск 01:21:47  384517
>>384516
Для заселения Марса надо чтобы колонисты не сдохли там в первом поколении и могли дать жизнеспособное потомство.
Аноним 25/02/18 Вск 01:24:07  384519
>>384517
В чем принципиальная проблема?
Аноним 25/02/18 Вск 01:27:46  384521
>>384519
Проблема в том чтобы приспособиться к новым условиям и не сдохнуть. Гугли естественный отбор.
Аноним 25/02/18 Вск 01:28:53  384522
>>384517
Никто рожать детей на Марсе не будет - это эксперименты над человеком. Ни одна шлюха не пожертвует своей пиздой, маткой и ребенком лишь бы поставить эксперимент над собой. И не разрешат белые Лыцари будущего SJW общества. И даже если такой эксперимент проводить, то нужно следить, а следовательно на Марсе должен быть йоба мед.клиника. А это тоже далеко по времени.
Аноним 25/02/18 Вск 01:29:01  384523
>>384521
К пониженной гравитации разве что.
Больше упражнений и кальциевых супплементов.
Остальное все технически реализуемо - 1 атм 80% азот 20% кислород, вода, еда. Че еще надо?
Аноним 25/02/18 Вск 01:30:11  384524
>>384522
СЖВ рак поразил только запад (и немножко нас), есть надежда на китаёз. Может быть и индусов. Может объединятся и полетят в один конец под неодобрительное гудение бородатых баб и геев в лосинах.
Аноним 25/02/18 Вск 01:34:45  384525
>>384523
> К пониженной гравитации разве что.
Ага, вот мы миллиарды лет развивались от бактерий до хомо при одном жэ, а тут бах и просто перестраиваем всю свою суть под одну треть от этого.
> Больше упражнений
То есть вместе того чтобы заниматься делом колонисты будут качаться.
> и кальциевых супплементов.
Это не панацея.
> Остальное все технически реализуемо - 1 атм 80% азот 20% кислород, вода, еда. Че еще надо?
Надо гравитация. Для размножения. Ну или придется выращивать человеков в искусственной матке помещенной в центрифугу. Это кстати более эффективный вариант чем проводить сомнительные эксперименты на людских самках.
Аноним 25/02/18 Вск 01:36:06  384526
>>384525
Решение найдется в любом случае, но для этого, к сожалению, надо чтоб жареный петух в жопу клюнул.
Аноним 25/02/18 Вск 01:42:04  384527
>>384526
К сожалению, если не будет глобального катаклизма, то при нынешних темпах развития технологий мы деградируем до состояния слабоумных животных.
Аноним 25/02/18 Вск 01:45:21  384528
photo-6591604.jpg (9Кб, 167x300)
>>384527
>мегаватт-часы электроэнергии на майнинг
>эмодзи в юникод
>гейфоны тоньше
>квоты для ниггеров
>и т.д. и т.п.
По-моему, мы уже хуже животных.
Аноним 25/02/18 Вск 01:47:30  384529
фаг.png (288Кб, 563x800)
до тех пор пока не полетим к Марсу где-нибудь в 2030х, я увижу столько ебучих разговоров о Марсе, столько ньюфагов будут задавать тупые повторяющиеся вопросы, что я наверное с ума сойду. И некоторые еще раньше меня это видят - одна и та же болтовня по тв и в интернете о Марсе, просто потому что ничего реального не делается. Если никто не летит, то и остается что чесать языками, относиться к проблеме полета на Марс как к какой-то метафизической проблеме, которая даже не связана с реальным космическим телом и касается лишь тела, которое существует для многих людей виртуально как набор косвенно заученных характеристик, потому что реально его даже в телескоп они его не видели. Вот если ты Марса не видел, но тебе говорят что он есть, все лишь говорят, что он есть, все лишь говорят о полете на Марс, то чем такие дискуссии отличаются от религиозных толков? Вот как люди говорят как они будут ебать 72 девственниц в раю, так и люди воображают о марсианских колониях, прекрасно зная что они ЛИЧНО туда не попадут, что лично они никакого марса не видели в жизни, для них он существует лишь как дискуссионная проблема, какой-то симулякр в языке, а не реальное тело. Чтобы Марс стал реальным к нему нужно полететь реально телом, но этого никто не делает, все лишь говорят, говорят, гворят, говорят, Марс существует лишь как проблема в спорах, а не как пункт назначения, не как географически-астрономическая точка в пространстве на которую замыкается маршрут какого-то человека.
Аноним 25/02/18 Вск 01:49:35  384530
>>384528
Ну, вообще, пока машины не начали производить и обслуживать сами себя то всё ещё не так плохо. Хотя современному человеку в развитых странах уже нет необходимости бороться за существование, строго говоря уже некоторым и ходить необязательно дальше нескольких сотен метров в день.
Аноним 25/02/18 Вск 01:50:52  384531
>>384529
Хороший тупой вопрос.
Ответ на него такой:
Нет.
Аноним 25/02/18 Вск 01:55:44  384534
>>384529
Тут надо начать вообще с задач. То есть зачем нам физически присутствовать на Марсе? То есть зачем там жить людям? Разве что сделать флаговтык. Всё.
Единственное зачем нам может быть нужен Марс это ресурсы. Но. Если у нас робототехника шагает семимильными шагами то намного проще качать оттуда ископаемые(я не знаю какие они там есть, впрочем можно и не в Марсе ковыряться, а например в поясе астероидов) при помощи автоматизированных фабрик которые без нашего непосредственного участия будут долбать поверхность и завозить на оскудневшую земляшку ништяки.
Аноним 25/02/18 Вск 01:59:25  384535
>>384534
>задач
А если вдруг по земле ебнет гигантский метеорит и убьет все живое? Ты об этом подумал? А? АА!?!?
Аноним 25/02/18 Вск 02:05:07  384536
>>384535
Прецедентов не было.
Аноним 25/02/18 Вск 02:07:10  384537
1517841741659.jpg (72Кб, 787x960)
>>384536
Из убивающих - Чиксулуб.
Из просто падающих - Челябинск.
Или надо чтобы при нас упал убивающий все живое метеорит? That kinda defeats the purpose...
Аноним 25/02/18 Вск 02:12:02  384538
>>384537
На нынешнем этапе развития такой метеорит мы заметим и успеем принять меры(то есть зарыться поглубже). После того как он ебнет выберется наружу (небольшая часть популяции, но неважно) и опять осваивать планету. То есть человечество выживет в любом случае.
Аноним 25/02/18 Вск 02:20:25  384541
image.png (246Кб, 600x366)
>>384538
Экологии пызда, дышать нечем, выживем, как же.
Аноним 25/02/18 Вск 02:38:41  384543
>>384541
> Экологии пызда
Ну, последний метеорит нашему племени пошел на пользу даже.
> дышать нечем
А это почему? Если допустим на всей планете выживет сто тысяч человек (я сомневаюсь что в бункеры можно упрятать больше) то уже имеющегося в атмосфере кислорода им хватит на многие поколения(это я примерно почувствовал, да). А дальше либо ишак сдохнет, либо шах, либо ходжа или приспособятся развив лёгкие большого размера например, либо, что более вероятно, засеять Землю растениям.
> выживем, как же.
Пережили же и мамонтов и саблезубов разных, хотя они здоровее.
Аноним 25/02/18 Вск 03:01:38  384545
>>384543
Кто приспособится, ты наркоман что ли? На приспособления надо не одно потомство выходцев из убежища, на это надо многие поколения.
А ядрен-батон убивающий все и вся попортит воздух в прямом смысле, вызовет глобальные тектонические реакции, ядерную зиму в купе со сгоранием всего что горит и выбросом всяких вулканических газов. Даже если атмосфера дышабельна - на дворе ядерная зима на многие десятилетия.
ИРЛ не фоллаут и не Метро 2013, чудные фантазии не покатят
Аноним 25/02/18 Вск 03:10:49  384546
>>384545
> ядрен батон
Так мы же за метеориты трем.
Да ладно, сколько там гигатонн ебанулось в мексиканский залив? И ничего, живём потихоньку.
А учитывая что мы шибко умные, шансы на выживание у нас очень большие. Это динозавры не знали что на них там чето ебнется а потом не смогли приспособиться и передохли все (оставив кстати своих потомков в виде птичек кстати, так что в некотором смысле они пережили сами себя).
Аноним 25/02/18 Вск 09:45:02  384550
>>384546
>Так мы же за метеориты трем.
А какая разница?
>Да ладно, сколько там гигатонн ебанулось в мексиканский залив? И ничего, живём потихоньку.
Да, прошло пару лет и починилось все.
Выжили, напомню, только мелкие хомячки-крыски, пожиравшие корешки, заныкавшись в говнище.
>в некотором смысле они пережили сами себя).
В некоторм смысле мы переживем сами себя в случае пиздеца. Те же крыски опыть эволюционируют в какое-нибудь гомо за сто мегалет. Только цивилизации людей пиздец.
Аноним 25/02/18 Вск 11:21:05  384560
Какая самая большая перегрузка из-за двигателей достигалась в пилотируемых полетах?
Аноним 25/02/18 Вск 12:12:04  384564
>>384538
>На нынешнем этапе развития такой метеорит мы заметим и успеем принять меры(то есть зарыться поглубже).
Вообще то нет, не успеем в некоторых случаях - например челябинский никто не заметил.
Аноним 25/02/18 Вск 12:37:50  384568
>>384560
Около 21-25 G в неудачном пуске "Союза 18-1" в 1975-м. Отказала вторая ступень (не смогла штатно отделиться, кажется), сработала САС. Вот во время спуска после отделения перегрузки и были сильнейшими, у экипажа на время остановились сердцебиения.
25/02/18 Вск 12:44:31  384569
>>384568
>во время спуска после отделения
Эээ может во время работы двигателей САС?
Аноним 25/02/18 Вск 12:59:54  384571
>>384568
Сколько в g? 20-30?
Аноним 25/02/18 Вск 13:27:34  384575
>>384569
>>384571
Именно во время спуска, так укзано во многих источниках. Они чудом остались живы еще и после приземлния, когда СА повис на склоне, зацепившись стропами парашюта за какой-то куст.
По силе перегрузок есть, опять же, разные цифры, но постоянно фигурируют данные о 21-23 или даже 26 G.
Аноним 25/02/18 Вск 13:55:05  384578
>>384575
>>384568
А в запланированных ситуациях?
Аноним 25/02/18 Вск 15:03:48  384593
>>384550
В том что фалаут объясняется радиацие. Если же будет просто метиорит уровня мезозой-кайнозойского. То умрут сразу только люди в месте падения. Остальные не умрут от лесных пожаров, т.к. люди и коровы едят рожь и пшено, а не ёлки и жёлуди.
>Выжили, напомню, только мелкие хомячки-крыски, пожиравшие корешки, заныкавшись в говнище.
Напомню, что под землёй существуют не только бункеры, но и метро. А человечество имеет предков всего от 10 000 человек.
>цивилизации людей пиздец.
Вот никогда не понимал кукареков, что мы все умрём. Мы можем выродится, заболеть супер оспой из будущего, заразится социальной болезнью. И то это сильно сомнительно. А тут какой-то метеорит должен почему-то убить всех людей и сразу. Тут либо всю биосферу сразу, либо человек выживет.


Вот тебе аналогия:
Сигнал тревоги. На военную часть 00007 летит ЯО. Все прыгают в бомбей. Мышь полёвка чуя суматоху в части, на всякий случай прыгает в свою нору 10 сантиметров от поверхности. ЯО прилетает акурат на один из 2х бомбеев. Тот анигилируется, в то же время 2 функционирует. Мышка полёвка оказывается достаточно далеко и везуча что бы не попасть под радиацию или взрыв. У военных же есть счётчик Гейгера, и они знают когда выходить и куда идти. Из за перебоев с продовольствием, солдат ставит мышеловку в поле, ловит эту полёвку и съедает её. Вопрос. Кто наиболее приспособленный?
Аноним 25/02/18 Вск 15:35:52  384598
>>384593
>метро
Блядь, ребенок, угомонись. Не поможет твое метро от extinction event, это не компуктерная игрушка.
Аноним 25/02/18 Вск 15:56:47  384600
>>384593
Хуя, уже выросло несколько поколений ебланов, которые вымрут при любом пуке, но они продолжают утверждать что все пучком. Мочало, ты полностью зависим, не будет жрат - ты вымрешь. Не будет спат - ты вымрешь. Не будет тепла - ты вымрешь. Самая главная проблема не в самом импакте, а в последствиях после него. Изменение климата, меньше солнца, кризис селхоза, меньше жрат и тд. Возможно химия и спасёт, но вопрос не проебутся ли технологии и не сойдут ли обезьяны с ума от катаклизмов остаётся открытым.
Аноним 25/02/18 Вск 16:21:16  384603
>>384600
Человечество какой только хуйни не переживало. Даже если скатимся к первобытному строю то для биологического вида это будет вин.
Аноним 25/02/18 Вск 16:22:39  384604
>>384603
Проблема в том, что при extinction event'е не скатимся в первобытный строй. Даже если твое любимое метро спасет от импакта (нет, наоборот все в метро погибнут сразу) то сто лет будет нечего есть, а в воде и воздухе говно.
Аноним 25/02/18 Вск 16:24:14  384605
>>384600
Чини детектор.
Еду можно найти. При отсутствии пищи в принципе, люди научились есть трупы и сказано же, что корешки остались, спать можно посменно, а проблему сохранения тепла мы решили ещё в каменном веке.
Люди не проебутся, в этом был вопрос. А что касается технологий, то мы уже прошли точку не возврата, сейчас каждый еблан знает из чего состоит ДВС и как сделать ЯО. Что бы утратить технологии, надо собственноручно уничтожить всех старше 12, что вряд ли произойдёт. Если ты про Москву, то да московские деграданты умрут первыми, но представь себе за МКАДом есть жизнь.
И это только в ваших фентези человек сходит с ума от переизбытка чувств. У человека есть способность адаптироватся к окружающим условиям. Быть может ты говоришь о далёком будущем, когда мы в конец деградируем к 4000 году. А на данный момент мы более приспособленные, чем все когда-либо живущие динозавры, включая ворон. И если сейчас жахнет человечество оправится, да это будет долго и хуёво, но к 2300-2400 году скорее всего выдут на сегоднешний уровень производства.
И да если ты говоришь о фименистках говоря что все сходят с ума. Боюсь тебя огорчить, не намерено такие ебланы не плодятся. Они живут на гранты и выполняют всё по указке грантадателя. Нет гранта -нет "феменизма"
>>384598
Если есть сила способная наебнуть все бункеры в мире, то этот катаклизм переживут только одноклеточные. О чём ты говоришь.

Тысячелетиями, как-то выживали, переживали массовые эпидемии, а тут когда и людей по больше и сил на порядки больше, вдруг всё на ебнётся. Или вы из тех кто считает что мы уже 7 цивилизация.
Аноним 25/02/18 Вск 16:25:22  384606
>>384605
>Если есть сила способная наебнуть все бункеры в мире, то этот катаклизм переживут только одноклеточные. О чём ты говоришь.
Именно об этом и говорю. Спастись от такого катаклизма можно только с помощью самодостаточной (по крайней мере длительное время) колонии на другой планете. Hint, hint.
Аноним 25/02/18 Вск 16:29:20  384607
>>384606
Это что за катаклизм такой?
Аноним 25/02/18 Вск 16:40:09  384608
>>384605
Падает в сибирь 9-и километровый астероид. Атмосферу и орбиту засерают расплавленная порода, кора, обломки, пепел, гарь, газы. Солнца не будет несколько лет. По всей планете пожары, которые нельзя потушить, кислотные дожди и отправленная атмосфера. Сельского хозяйства больше нет, экономики тоже.
Аноним 25/02/18 Вск 16:42:21  384609
>>384606
Какова вероятность этого катаклизма?
Произойдёт ли она ближайшие 100 000 лет?
Нет?
Тогда и парится об этом не стоит? Какой ближайший объект могущий в это церера, палада?
Что там по рен-тв показывают апофис? 300 метров, это ваша машина судного дня?
Да это дохуя для всяких правительств и кучи народу. Но с точки зрения вида и цивилизации это хуйня.

А раз через и через 100 000 лет не ебанёт, то и смысла торопится не особо много. Через 100 000 лет и ресурсы появятся и мотив может найдётся. Пока этого мотива нет. Экономические риски не оправданы. Точно так же можно ожидать что БАК наплодит чёрных дыр и сожрёт землю. Или что эбола приобретёт разум и начнёт играть в симулятор пандемий. Или боятся что на тебя лично упадёт спутник, и по этому не выходить из дома. Или даже более точная аналогия, наоборот, что на тебя упадёт спутник, когда ты сидишь дома, по этому ты спишь по 4 часа и скорее спешишь на улицу.
Аноним 25/02/18 Вск 16:46:47  384610
Чем круглые солнечные панели отличаются от прямоугольных?
Аноним 25/02/18 Вск 16:47:18  384611
>>384609
>Какова вероятность этого катаклизма?
Ненулевая. Каждый день мы играем ва-банк.
>Произойдёт ли она ближайшие 100 000 лет?
Может даже метеорита для этого ждать не придется, за это время супервулкан ебанет и настанет ядерная зима, массовый голод и половина человечества просто от голода помрет и хуй знает еще сколько в войнах.
Аноним 25/02/18 Вск 16:48:13  384612
>>384610
Формой.
Они раскладываются по другому. Какие удобней сложить, так и складывают.
Аноним 25/02/18 Вск 16:55:16  384613
>>384608
Есть охота, собирательств и автомат калашникова охраняющие запасы еды. Через 20 лет найдётся или создастся участки без химии, где можно растить пищу. Злые газы давно уже осядут, не проникнув через противогаз и двери бункера. человечество немного поредеет из-за конфликтов и голода, однако сухая библиотека содержащая тысячи тысяч школьных учебников почему-то сохранилось.
Основной упор ты выставляешь на то что нет земли. Но она везде есть, обезаразь существующую или просто вытащи из складированных пакетов из цветочного магазина. В конце концов, люди не такие идиоты, что видя что на другой стороне земли упал метеорит, сидеть и ждать пока до тебя долетят химические облака. А экономика это лишь средство распределения. Если есть ресурсы, то и экономика приложится. Катаклизм же ударяет по людским и продовольственным ресурсам. Их можно как-то и защитить.
Аноним 25/02/18 Вск 17:10:41  384617
>>384611
Ближайшие 1000 лет ни один супервулкан не взорвётся. Ещё рано. Да и сейчас разрабатываются методы по предотвращению последствий.
Ненулевая вероятность, что вострубят 7 ангелов и тебя будут судить за то что ты кошку гладил против шерсти. Ненулевая вероятность что ты оказывается шварцнегер и ты герой фильма "Последний Киногерой" или то что фильм Матрица основан на реальных событиях с тобой же на главных ролях.
Но обращать на эти астрономически малые шансы глупо, и контрпродуктивно. Ты скорее погибнешь если ты будешь вкладывать ресурсы в какую-нибудь хуйню типа марса, нежели если вкладываться в реальный прогресс заключающийся по большей части в материаловедении и отработке технических решений, и через 100-1000 лет уже иметь возможность и мотив уже по флагофтыкать.
Ну прилетишь на марс, ну съебёшь от туда через неделю. Тебя будут прославалять "от какой молодец, обнаружил гидрохуйзин в коре марса", а через 100 лет в рабочем порядке какая-нибудь тётя клара будет сканировать местность и в поисках нормальных ресурсов, а не этот ваш устаревший гидрохуйзин. Итого твоя работа пошла на смарку. Точно так же как и работа аполлонов, хотевшим там сделать базу. Но увы это было контрпродуктивно и вообще сделанное лишь из целей пристижа, а не чего ещё.
Аноним 25/02/18 Вск 17:21:38  384619
image.png (599Кб, 700x546)
>>384617
Аноним 25/02/18 Вск 17:53:59  384622
>>384617
Чванливый хуесос, такие проекты как полеты на Луну или Марс дают охуенный профит в долговременной перспективе. Тот же полет на Луну окупился у американцев на раз-два. И это только экономическая составляющая. А есть еще надличностная цель необходимая для развития человечества. Так что втыкай себе флаг в жопу и сиди тихо.
Аноним 25/02/18 Вск 18:30:49  384629
15142256019550.jpg (52Кб, 571x448)
>>384622
> Тот же полет на Луну окупился у американцев на раз-два.
Чего?
Аноним 25/02/18 Вск 18:58:55  384632
>>384629
Не тупи.
Аноним 25/02/18 Вск 19:00:16  384633
>>384632
Я не туплю а хочу узнать.
Аноним 25/02/18 Вск 21:25:56  384645
Как же колонию на Титане хочется...
Аноним 25/02/18 Вск 21:31:36  384648
>>384645
Как в фиаско?
Аноним 25/02/18 Вск 21:54:06  384652
>>384645
И чтобы там люди в ватниках летали...
Аноним 25/02/18 Вск 21:55:23  384653
Почему на Венеру ничего не пускают? Только 1 японский мелкоспутник и все. Я понимаю что на поверхности там хуже чем в жопе, но на орбиту вроде заебок пускать, Венера же рядом.
Аноним 25/02/18 Вск 21:55:47  384654
>>384645
Сап двач. Дело в том, что есть одна колония...
Аноним 25/02/18 Вск 22:01:10  384655
>>384653
Потому что там нихуя нет. Совьет юнион наглядно показал, что на поверхности боль, слёзы и пиздец, с орбиты только пялься в облака или свети радаром.
Пиарастической кококолонизация-ценности как Марс не несёт - всё равно что жить в ванне с кислотой.
Пиарастической ценности а-вдругтам-жизнь-покпок тоже не несёт - там пиздец.
Такого научного интереса как отдалённые куски говна не несёт - она близко, аппаратов туда накидали достаточно, чтобы понять, что там уныние. Намного интереснее сейчас тратить деньги на что-нибудь менее изученное.
Аноним 25/02/18 Вск 22:01:16  384656
>>384652
На ватниках
Аноним 25/02/18 Вск 22:11:13  384657
>>384655
>всё равно что жить в ванне с кислотой.
Не так уж все и кислотно если что. На поверхности СО2 в сверхкритическом состоянии. Серная кислота только в виде облаков высоко витает.
Аноним 25/02/18 Вск 22:22:32  384658
>>384629
Политические дивиденты, инфраструктура и технологии. Я бы сказал, окупился с головой.
Аноним 25/02/18 Вск 22:40:56  384659
>>384658
Это имеет какое-то числительное значение?
Аноним 25/02/18 Вск 22:41:44  384660
>>384659
Да. Примерно двадцать.
Аноним 25/02/18 Вск 22:44:16  384661
>>384660
Двадцать старых ботинок?
Аноним 25/02/18 Вск 23:00:11  384662
>>384534
>зачем нам физически присутствовать на Марсе?
ПРОСЫПАЕТСЯ КАЖДЫЙ ДЕНЬ С МЕЧТОЙ!!!1 МЕЧТА!11 ЖИЗНЬ БЕЗ МЕЧТЫ ПУСТА!!11
Мне вообще похуй, что ты там думаешь, рачок. Можешь ещё статей про мечту накатать, неюоходцесектант.
Аноним 25/02/18 Вск 23:02:50  384663
>>384662
Ты уверен, что ссылкой не ошибся?
Аноним 25/02/18 Вск 23:03:30  384665
>>384662
Ты пьяный?
Аноним 25/02/18 Вск 23:05:47  384668
>>384665
В вечер воскресенья на двачах каждый второй пьяный или объёбанный.
Аноним 25/02/18 Вск 23:06:43  384669
>>384668
Но ведь школьникам мама не разрешает.
Аноним 25/02/18 Вск 23:09:49  384670
>>384663
>>384665
Я про то, что задач для колонии на нём Марса нет, а сектанты пишут про мечту у сами знаете кого, мол это его перводвижитель. И смешно и больно.
Аноним 25/02/18 Вск 23:12:43  384671
>>384670
Лично я тот на чей пост ты ответил. Но я и написал что людям на Марсе людям и не обязательно присутствовать. Если надо будет качать оттуда ресурсы то с этим справятся автоматические заводы. Ну, для гарантии можно посадить там пару сычей.
Аноним 25/02/18 Вск 23:18:55  384672
>>384671
>Ну, для гарантии можно посадить там пару сычей.
Напомнило:
https://www.youtube.com/watch?v=52Gg9CqhbP8
Аноним 25/02/18 Вск 23:45:55  384677
>>384633
Например «тефлон».
Аноним 25/02/18 Вск 23:47:32  384678
На какой максимальной высоте полетит ракета с ядерной боеголовкой из Москвы в Вашингтон?
Аноним 25/02/18 Вск 23:56:59  384679
>>384677
И хули твой тефлон? Его изобрели ещё в 38.
Аноним 25/02/18 Вск 23:57:50  384680
>>384678
Из Москвы не полетит.
Аноним 26/02/18 Пнд 00:05:06  384682
>>384678
На любой. Зависит от ТТХ конкретной ракеты.
В среднем около 1000-1200 км, но опять же, всё зависит от задач.
Некоторые (пресловутая "Булава", например) разрабатывают так, чтобы они летели пониже. Но если надо, можно наоборот, соорудить ёбу, которая доставит груз потенциальному другу через облёт Луны.
Или сделать орбитальную боеголовку, там вообще смешно получается.

Ну и да, из Москвы с ядерной боеголовкой разве что какая-нибудь суровая советская ракета ПРО могла бы вылететь, а не межконтиненталка.
Аноним 26/02/18 Пнд 00:05:16  384683
>>384622
>>384658
Всё это можно сделать и без космоса. В 1 и 2 это выступает как повод, причина которого поднасрать СССРу. И это не технологии появляются в космической отрасле, а космическая отрсль использует технологии, которые и без того производятся в специальных лабах. И вместо того чтобы давать деньги на какой-то влаговтык, куда рациональнее направить их в эти лабы непосредственно.
Вот мы сейчас повышаем автономность, что бы в будущем иметь возможности в колонии. Но пока что это не более чем прототипы существующие 1 экземпляре. И до тех пор пока этого не достигнем, когда мы сможем с комфортом полететь на луну в молодости и умереть там от старости, вот тогда да, а на ближайшие сотни лет мы застряли на земле, и на марсе нам нужно исключительно пофлаговтыкать.
Аноним 26/02/18 Пнд 00:17:37  384684
space bond.webm (8889Кб, 640x272, 00:02:03)
А ну ещё державы хотят в космосе воевать.
>>384683
Аноним 26/02/18 Пнд 00:20:20  384685
>>384684
>сражаться пихотой в космосе в зоне прямой видимости
>когда техника за многие километры видит противника и способна нанести урон
>когда достаточно высыпать ведро гаек
Аноним 26/02/18 Пнд 00:26:03  384686
STS-51-FClip318.jpg (138Кб, 903x603)
>>384684
Как же я обдристался с этого видео. Теперь это засмеялся-на орбите остался тред?
Аноним 26/02/18 Пнд 02:03:07  384689
>>384128
Во-первых бумп.

Во-вторых можно ли определить возраст звезды не по составу? Скажем какая-то звезда зародилась акурат после бигбенга, а затем на неё налетело облако всякой хуйни больно металичной и ученые такие: " да она же малая" Хотя она самая старая.
Аноним 26/02/18 Пнд 02:53:48  384695
>>384128
>>384689
>>383889
>Остыв на 10 градусов до нежизнепригодных 26 градусов цельсия он все равно будет излучать 460 ватт тепла.
Аноним 26/02/18 Пнд 06:37:49  384723
>>384564
Челябинский и до земли не долетел, смекаешь?
Аноним 26/02/18 Пнд 08:53:45  384736
Могут ли рачки с Энцелада вылетать в плюмах? Касинни не проверял?
Аноним 26/02/18 Пнд 09:19:34  384740
>>384736
Могут. Касинни нечем было это проверять.
Аноним 26/02/18 Пнд 10:43:31  384760
>>384683
>куда рациональнее направить их в эти лабы непосредственно
Ох вау, каков идеалист однако.
Аноним 26/02/18 Пнд 11:19:40  384764
>>384683
Твое мнение очень важно для нас. Пожалуйста, держи нас в курсе.
Аноним 26/02/18 Пнд 11:27:15  384766
>>384529
Создание альтернативного технологичного общества, где каждый человек нужен и занимается полезными вещами, а не вот это все вокруг. И желательно не пускать туда политиков и мракобесов.
Аноним 26/02/18 Пнд 11:36:34  384769
Adeptusmecanics.jpg (27Кб, 400x407)
>>384766
Аноним 26/02/18 Пнд 18:37:25  384879
>>384760
Так-то да, деньги слабо помогут этим лабам, но они хотя бы купят оборудование что сократит время исследований. А флаговтык лишь покажет что твоя держава самая первая. Холодная война закончилась, и это теперь бессмысленно.
Аноним 26/02/18 Пнд 18:54:16  384887
>>384879
Ещё один конченый...
Аноним 26/02/18 Пнд 21:09:26  384923
Как ДСГ снабжать будут и людей менять? Каждый раз на СЛС это же разорение.
Аноним 26/02/18 Пнд 21:26:56  384926
>>384923
>ДСГ
Это роботизированная КПП у вага жи.

мимо из автача
Аноним 26/02/18 Пнд 21:28:58  384927
>>384923
Для снабжения есть ФХ, Вулкан, Нью Глен. У двух последних будет вторая ступень на метане/водороде, у первого тоже возможно.
Других вариантов пулять обезьян к Луне нет. Может быть когда-то у кого-то чего-то.
Аноним 26/02/18 Пнд 21:29:36  384928
>>384927
Не забывай про Российские ракеты.
Аноним 26/02/18 Пнд 21:32:45  384929
>>384928
Насе будут интересны только свои частники. Да и гептиловоз может вообще развалиться к хуям к 25. А новых не завезут.
Аноним 26/02/18 Пнд 21:33:34  384930
>>384927
Лысый же хочет людей туда пулять, не?
>>384928
Как там ход работ по федерации и рн?
Аноним 26/02/18 Пнд 21:36:32  384931
>>384930
Лысый хочет пулять на 100 км. И хочет сделать лунный модуль на деньги насы. Хитрый лысый.
Аноним 26/02/18 Пнд 21:38:18  384932
>>384931
Это разные ракеты.
Аноним 26/02/18 Пнд 21:39:53  384933
>>384932
Люди на Глене не планируются на данный период времени, только Шепард.
Аноним 26/02/18 Пнд 21:45:12  384934
>>384933
В ближайшие несколько лет планируется пуск людей в облет луны, ели не ошибаюсь.
Аноним 26/02/18 Пнд 21:49:01  384935
>>384934
Это чья-то фантазия. Может быть когда доделают глен, начнут его регулярно пускать и дорабатывать, может быть тогда. Вообщем это настолько неопределённо, что серьёзно воспринимать не стоит.
Аноним 26/02/18 Пнд 21:49:02  384936
>>384934
Элон Маск в этом году общеал облет луны
Аноним 26/02/18 Пнд 21:58:53  384938
>>384936
Этот облёт находится там же, где наша лунная база, которую в 2015 построили.
Аноним 26/02/18 Пнд 22:08:20  384940
>>384935
Ну а как же голубая луна?
Аноним 26/02/18 Пнд 22:19:11  384941
Есть варианты электрических двигателей на гептиле?
Аноним 26/02/18 Пнд 22:35:40  384948
>>384695
Я спросил не про излучение, а про охлаждение через испарение влаги с кожи.
Аноним 26/02/18 Пнд 22:49:15  384950
>>384948
Еще тепла унесет, конечно. Но какая разница, если излучения достаточно?
Аноним 27/02/18 Втр 00:27:02  384961
>>384682
Ого, 1000 км это оче высоко. Далеко взлетать, далеко вниз потом лететь.
Аноним 27/02/18 Втр 00:28:25  384962
>>384961
Я сам удивляюсь, зачем так высоко?
Аноним 27/02/18 Втр 01:20:31  384963
>>384962
Мир посмотреть, себя показать.
Я думаю это чтобы большую тёплую ламповую ракету было сложнее спалить и/или уебошить на этапе работы двигателей. Но я не специалист с мировым уровнем из /вм, так что могу ошибаться.
Аноним 27/02/18 Втр 01:45:32  384964
>>384962
Потому что хуярят по баллистической траектории.
Аноним 27/02/18 Втр 02:00:03  384965
trajectories[1].gif (10Кб, 562x251)
>>384962
Ракета летит так высоко, потому что это оптимальная траектория, если сделать траекторию более пологой или более крутой, то для того же расстояния будет меньше забрасываемый вес.

Да и угол входа в атмосферу тоже важен, чем он выше, тем больше перегрузки и сильнее припекает, но и перехватить боеголовку на конечном этапе становится сложнее.
Аноним 27/02/18 Втр 02:29:58  384967
>>384964
Любая траектория баллистическая, при выключенном двигателе. На самом деле там эллипс, но при таких параметрах можно упростить до параболы.
Аноним 27/02/18 Втр 02:38:48  384968
>>384682
>из Москвы с ядерной боеголовкой разве что какая-нибудь суровая советская ракета ПРО могла бы вылететь, а не межконтиненталка.
Чойто? В Козельске дивизия РВСН стоит, Плесецк опять же. Какая разница, откуда через полюс пускать.
Аноним 27/02/18 Втр 02:45:57  384969
11697-1445677730.jpg (35Кб, 640x519)
>>383889
>Остыв на 10 градусов до нежизнепригодных 26 градусов цельсия он все равно будет излучать 460 ватт тепла.
Только если он абсолютно черный.
Аноним 27/02/18 Втр 06:20:42  384974
>>384968
Чтобы снизить приоритетность и не класть все яйца в одну корзину.
Аноним 27/02/18 Втр 07:36:23  384975
>>384968
>Чойто?
В первую очередь потому, что в Москве и окрестностях нет межконтиненталок, только ПРО.
Аноним 27/02/18 Втр 08:13:33  384976
>>384965
Но ведь выше 100 км атмосфера почти не влияет для полета в одну точку и можно лететь пониже
Аноним 27/02/18 Втр 09:40:12  384984
Есть где нибудь календарик и прибытиями и отбытиями АМС, зондов и прочей хуйни на ближайшие пару лет?
Аноним 27/02/18 Втр 09:49:16  384985
>>384975
>https://ru.wikipedia.org/wiki/28-%D1%8F_%D0%B3%D0%B2%D0%B0%D1%80%D0%B4%D0%B5%D0%B9%D1%81%D0%BA%D0%B0%D1%8F_%D1%80%D0%B0%D0%BA%D0%B5%D1%82%D0%BD%D0%B0%D1%8F_%D0%B4%D0%B8%D0%B2%D0%B8%D0%B7%D0%B8%D1%8F
200 км от Москвы тебе не окрестности?
Аноним 27/02/18 Втр 13:39:37  385016
>>384965
45° оптимальный угол, физика 7 класс
Аноним 27/02/18 Втр 13:45:19  385017
>>385016
Но у тебя ебически заправленная ракета, тебе бюджет позволяет в нее впихнуть кучу соляры чтоб она не вырубая движок херачила вплоть до пентагона. Я утрирую, но реально - нахуя "оптимальные" по энергозатратам траектории, когда тебе надо как можно быстрее?
Аноним 27/02/18 Втр 13:48:10  385019
>>384984
> Гугел
> список амс
> список космических телескопов
https://ru.m.wikipedia.org/wiki/Список_межпланетных_космических_аппаратов
https://ru.m.wikipedia.org/wiki/Список_космических_телескопов
Аноним 27/02/18 Втр 13:50:22  385022
>>385017
> бюджет позволяет в нее впихнуть кучу соляры
Так просто тут нихуя не масштабируется.
Аноним 27/02/18 Втр 13:57:26  385024
>>385017
Надо не как можно быстрее.
Скорость вообще примерно одинаковая всегда будет.
Разгонишься быстрее - тупо выйдешь на орбиту.
Дело в надежности, стоимости, простоте производства.

Вообще современные мбр маленькие, ибо забрасываемый вес не больше тонны.

Вот кстати и причина.
Раньше забрасываемый вес был очень важен, ибо там всегда было напихано 10+ мегатонн.

Да и сейчас, возьмем РГЧ.
Ты можешь распидорасить 6 объектов или только 3.
Что выберешь?
Аноним 27/02/18 Втр 14:13:23  385026
>>385019
Там ебаный список и хуй отличишь реальные миссии от бумажных.
Аноним 27/02/18 Втр 14:23:45  385031
>>385024
У нас есть мобильные ракетные установки у которых несколько боеголовок. Чето мегатонну вроде и выйдет если не больше.
Почему в шахту нельзя поставить ракету помощнее и не полететь быстрее с теми же мегатоннами?
Аноним 27/02/18 Втр 17:43:40  385074
Гетил и гидразин это одно и то же?
Аноним 27/02/18 Втр 18:12:52  385076
>>385074
Нет
Аноним 27/02/18 Втр 18:48:07  385079
>>385074
Гептил = коммерческое название монометилгидразина (ММГ). Также используется как жаргонное название для 1,1-диметилгидразина (НДМГ).

Гидразин = отдельное соединение, являющееся основой для метилгидразинов. Иногда используется как жаргонное обобщение для топлив на метилгидразиновой основе.
Аноним 27/02/18 Втр 19:22:39  385090
>>384985
>200 км от Москвы тебе не окрестности?
Вот честно — нет, не окрестности.
Километров 10, с натяжкой 20, с охуенной натяжкой с сопутствующим разрывом совы — 30, но не 200, нет.
Но это уже не тематика тхреада.
Аноним 27/02/18 Втр 21:30:23  385121
Посоветуйте программку где я могу смотреть текущие положения космических обьектов (например той же Теслы). Желательно с 3D моделями планет солнечной системы. И инфой о скорости обьекта и его массе. Спасибо заранее.
Аноним 27/02/18 Втр 21:44:49  385122
Здорово, космоёбы!

У меня к вам большой вопрос

Крч. Такая делега, у тебя есть телепорт на овер дохуя км, и телескоп которым можно рассмотреть сисечки школьницы находясь на Альфа Центавре.

Ты делаешь мгновенный телепорт на ебаную центавру, и смотришь на землю.

Внимание вопрос: что ты увидишь в телескопе? Ты можешь увидеть там самого себя заходящего в телепорт?
Аноним 27/02/18 Втр 22:07:35  385124
>>385122
>Внимание вопрос: что ты увидишь в телескопе?

Майдан.
Аноним 27/02/18 Втр 22:12:04  385125
>>385121
https://ru.wikipedia.org/wiki/Celestia

>Внимание вопрос: что ты увидишь в телескопе?
Предположим, на постройку телепорта ты купил участок земли девственного леса и за 4,5 года построил телепорт.
ТАк вот, прыгнув в телепорт с телескопом подмышкой на Альфа-Центавру и посмотрев на Землю ты увидишь себя, с топором посредине ебучего леса.
Аноним 27/02/18 Втр 22:12:55  385126
>>385122
>>385125

Аноним 27/02/18 Втр 22:18:34  385127
>>385122
>>385125
>что ты увидишь
Анон ещё валит деревья с одного удара, Крым ещё украинский, Америкой ещё правит негр, "Тесла" Маска ещё ездит по земле, Двач ещё торт... Нахуя улетал?..
Аноним 27/02/18 Втр 22:20:34  385128
Правда что на Титане воняет говном?
Аноним 27/02/18 Втр 22:23:57  385130
>>385128
Нет, не правда.
Аноним 27/02/18 Втр 22:28:10  385131
>>385125
Пиздато. Если Вселенная бесконечна, и люди изобретут всю эту байду, то можно будет прыгать овер дохуя далеко, и фапать на сиськи реальной Клеопатры
Аноним 27/02/18 Втр 22:29:06  385132
>>385128
но, если вокруг тебя воняет говном, это не значит что ты на Титане
Аноним 27/02/18 Втр 22:39:04  385134
>>385079
>Гептил = коммерческое название монометилгидразина (ММГ)
Нет, это несимметричный диметилгидразин.
Аноним 27/02/18 Втр 22:40:29  385135
>>385122
Да, сможешь. Ты сможешь видеть себя вплоть до момента телепортации целых четыре года.
Аноним 27/02/18 Втр 22:43:03  385136
>>385128
Там нет меркаптана и уж подавно продуктов работы микроорганизмов, а сам метан не пахнет.
Серы там всего ничего, а за вонь зачастую влияет именно она. Чистые углеродные соединения никак не пахнут в основном

>>385127
>Двач ещё торт.
Вот врать-то зачем?
Аноним 27/02/18 Втр 22:49:41  385137
>>385134
Во всех справочниках это именно ММГ, НДМГ приводится отдельно. В целом это спор о терминах
Аноним 27/02/18 Втр 22:52:56  385138
>>385137
Спор о терминах indeed. Но когда говорят о слове "гептил" подразумевают НДМГ в 90% случаев.
Вот ни разу не слышал про гептил на Аполлонах, например, а там как раз ММГ.
Аноним 27/02/18 Втр 22:54:20  385139
>>385138
Я слышал про гептилки омс шаттлов
Аноним 27/02/18 Втр 22:55:17  385140
>>385139
Было бы неплохо увидеть ссылку на статью. Потому, что личные анекдоты ничего не значат.
Аноним 27/02/18 Втр 22:56:07  385141
>>385136
>меркаптан
>Серы там всего ничего, а за вонь зачастую влияет именно она.
Ни при чем. Тот самый запах обеспечивает скатол.
Аноним 27/02/18 Втр 22:57:41  385142
>>383666
Платина.
Аноним 27/02/18 Втр 22:59:26  385144
brainlet 1.png (126Кб, 601x508)
>>385141
Кто бы мог подумать. Спасибо за просвещение.
Аноним 27/02/18 Втр 23:15:47  385149
>>385140
>There are 64 ac -motor-operated valve actuators in the OMS/RCS nitrogen tetroxide and monomethyl hydrazine propellant systems.

https://spaceflight.nasa.gov/shuttle/reference/shutref/orbiter/oms/storage.html

Опять приплываем к спору о терминах, где гептилом называют любое производное гидразина, используемое в качестве ракетного топлива. Сам гептил - это гептан с проебаным атомом водорода. Поэтому требовать некую ссылку на некую статью тут вряд ли уместно. Это скорее устоявшийся термин, нарицательное название. Типа как называть любое копировальное устройство ксероксом.
Аноним 27/02/18 Втр 23:24:23  385151
>>385149
А где там было слово "гептил"-то?
Аноним 27/02/18 Втр 23:56:01  385154
>>385151
>А где там было слово "гептил"-то?
Ты не понял, ты не найдешь слово "гептил" где-нибудь в "приличном месте", относительно метилгидразина, или НДМГ, потому как гептил к ним отношения вообще не имеет. Это совсем другое вещество. Оно досталось этим соединениям от военных в пору высокой секретности ракетных программ. Так же как амил совсем не N2O4. Гептилом можно хоть жопу ужа называть, если хочется.
Аноним 28/02/18 Срд 00:00:40  385156
>>385154
Так ты же утверждаешь про название "гептил", но по ходу выходит что это твои личные истории и байки, на деле в интернете никто так ММГ не называет, получается?
Зачем тогда нам его называть гептилом, почему бы нам не продолжать называть Гептилом НДМГ и амилом Н2О4?
Аноним 28/02/18 Срд 00:19:58  385160
>>385156
>в интернете никто так ММГ не называет, получается?
Получается, что в википедии, например, называет. И что теперь? Нахуй пойдешь, или останешься еще поспорить про гвоно и мочу?
Аноним 28/02/18 Срд 00:54:02  385166
image.png (122Кб, 1713x702)
>>385160
>Получается, что в википедии, например, называет.
Беспруфная правка это... Гм. Это беспруфная правка.
Есть у тебя еще источники?
> Нахуй пойдешь, или останешься еще поспорить про гвоно и мочу?
Если ты хочешь, чтобы я повел себя подобающе сайту, то это не ко мне, я в спейсаче веду себя с уважением к собеседнику.
Ты заявил, что гептилом зовут не только НДМГ, я был уверен что только НДМГ. Есть ли у тебя ссылки идоказательства? Никто из моих знакомых не слышал чтобы не НДМГ а другие гидразины называли гептилом. Если просветишь, то будет хорошо. Если нет - то это просто твой личный анекдот и не навязывай его другим.
Аноним 28/02/18 Срд 01:22:17  385167
>>385166
-Никто не называет.
-Называют, как хотят, кто хочет.
-Где пруфы?
-Ну вот.
-Ваши пруфы не пруфы.

Да катись к хуям, говноед.
Аноним 28/02/18 Срд 01:25:07  385168
>>385167
Но ведь аноним с вики действительно так себе пруф.
Аноним 28/02/18 Срд 01:27:40  385169
>>385167
Ты же не привел ни одной ссылки. Ты сказал, что на вики так называют, я показал ,что там анон сделал правку год назд.
Это правда доказательство для тебя?
Аноним 28/02/18 Срд 01:28:17  385170
>>385167
Если я назову метилгидразин мочой слона, то это тоже будет означать, что кто-то в интернете его так называет.

Мимо
Аноним 28/02/18 Срд 01:31:06  385171
>>385169
-Никто не называет
-Ну вот же, называют
-Никто не называет, даже правку в вики кто-то делал год назад. Наверно тот, кто не называет.

Ты понимаешь, что ты шизик?
Аноним 28/02/18 Срд 01:49:37  385172
yoba facepalm.jpg (42Кб, 604x579)
>>385171
Ты реально основываешь свое мировоззрение на неподтвержденных правках?
Аноним 28/02/18 Срд 02:17:49  385173
>>385171
Вообще то на вики сказано, что это не гептил.
Аноним 28/02/18 Срд 02:26:33  385174
>>385166
Насколько я знаю из школьной химии, никто не называет гептилом, гептолом, что-то отличное от соединения углерода с водородом, где приставка гепт обозначает что там 7 атомов углерода. А нет я проебался Несимметричный диметилгидразин его ещё называют.
Н-N-H
|
CH3-N-CH3

Метилгидразин совсем другая хуйта даже с не похожей формулой.
Аноним 28/02/18 Срд 02:28:04  385175
>>385174
Второй ряд соединяет две буквы N, сорян за проёб
H
|
N-H
|
N-CH3
|
CH3

Без учёта углов, конечно.
Аноним 28/02/18 Срд 02:31:51  385176
>>385174
>>385175
Мне вот интересно, есть ли где называют гептилом что-то помимо НДМГ?
Аноним 28/02/18 Срд 03:28:30  385179
Сап двач, есть одна дыра...
И не дает мне покоя форма того что собственно она из себя представляет особенно после попыток интерстеллара это визуализировать.
Суть заморочки в том что пространство-время не зря через тире пишут, получается что чем быстрее ты двигаешся в пространстве тем медленнее ты двигаешся во времени.
Выходит что при преодолении скорости света движение во времени должно прекратится совсем для внешнего наблюдателя.
Получается что все что достигает горизонта событий этот самый горизонт пересекает уже на скорости света. Все что ушло за горизонт в нашем понимании перестает двигаться так как время для этого объекта остановилось. Тут воображение взяло свой предел, получилась не плотная сфера а этакий пузырь стенки которого это замершая во времени материя которая всю оставшуюся вечность будет стремится к сингулярности в эпицентре всей этой дичи.

Как перестать об этом думать?
Аноним 28/02/18 Срд 06:38:01  385192
>>385179
>Получается что все что достигает горизонта событий этот самый горизонт пересекает уже на скорости света
Неверный вывод.
Аноним 28/02/18 Срд 09:37:38  385214
Существует ли космическая сверхманевренность?
Аноним 28/02/18 Срд 11:02:59  385220
>>385214
Нет, т.к. существование инертной массы у твоей мамки и соответствующие релятивистские эффекты при приближении скорости к световой никто не отменял.
Аноним 28/02/18 Срд 11:08:06  385222
>>385135
А еще он сможет подать самому себе сигнал не прыгать и не прыгнет, что вызовет парадокс. Поэтому телепортация невозможна.
Аноним 28/02/18 Срд 11:58:57  385235
>>385222
Не может же.
Аноним 28/02/18 Срд 12:30:41  385255
Стикер (63Кб, 500x500)
>>385125
То есть! Мы сейчас с земли смотрим на какую-то звезду в миллионе световых лет от нас и видим какой она была миллион лет назад, так? То есть сейчас там могут быть какие-то цивилизации, но мы их не видим, потому что свет от нас не дошел, так? Тоесть мы тут все смотрим в космос и видим только прошлое, так?
Аноним 28/02/18 Срд 12:32:32  385257
>>385255
Свет от НИХ не дошел до нас.
Да, так и есть.
Щас ты узнаешь про световой конус и поймешь, что внезапно ты ВСЕГДА видишь прошлое.
Аноним 28/02/18 Срд 12:46:38  385272
Стикер (0Кб, 512x512)
>>385257
>Да, так и есть.
Ну тоесть вокруг нас это все измениться должно было все, так? И мы никак не можем увидеть картинку что именно сейчас происходит за миллиард световых лет от нас. И излучение же тоже не быстрее скорости света летит. Пиздец кароче.
Аноним 28/02/18 Срд 12:51:08  385275
>>385272
>что именно сейчас происходит за миллиард световых лет от нас
Какая тебе нахуй разница что там именно сейчас происходит? Любые последствия происходящего именно сейчас до тебя дойдут не быстрее скорости света.
Аноним 28/02/18 Срд 12:54:42  385278
Стикер (127Кб, 500x500)
>>385275
Ну пиздец ваще, там уже ничего может и не быть совсем, а мы смотрим. Нахуя мы на миллиард лет назад смотрим?
Аноним 28/02/18 Срд 12:56:21  385280
>>385278
Хуя себе. Природа дала машину времени, можно посмотреть, что было хуй знает когда, а ты ебало отворачиваешь?
Аноним 28/02/18 Срд 12:56:25  385281
>>385278
>>385272
Так устроена Вселенная в которой мы живем.
Я тоже удивился в свое время.
Но тебе надо это понять и принять.
Аноним 28/02/18 Срд 14:05:51  385298
>>385278
Ну, таким образом мы имеет картину развития в течении миллиардов лет, так что это даже заебись. Вот в земную историю мы не можем заглянуть и поэтому принимаем писанину хуй пойми кого за правду, а в космосе мы можем напрямую видеть прошлое.
Аноним 28/02/18 Срд 14:07:48  385300
>>385179
>получилась не плотная сфера а этакий пузырь стенки которого это замершая во времени материя которая всю оставшуюся вечность будет стремится к сингулярности в эпицентре всей этой дичи.
Не обязательно, материя может распадаться на элементарные частицы и испаряться обратно в околодыровое пространство. Гугли излучение Хокинга.
Аноним 28/02/18 Срд 14:16:38  385303
>>385298
>напрямую видеть прошлое.
Может поэтому мы никакой внеземной жизни и не видим? Она сейчас может быть и везде, а мы просто не видим из-за вот этой всей хуйни.
Аноним 28/02/18 Срд 14:18:12  385304
>>385179
>Все что ушло за горизонт в нашем понимании перестает двигаться так как время для этого объекта остановилось.
Время остановилось для внешнего наблюдателя. Для тебя то все ок, ты в черную дыру летишь.
Аноним 28/02/18 Срд 14:27:04  385306
>>383656 (OP)
сохраните, если кому фото понадобятся
https://images.nasa.gov/
Аноним 28/02/18 Срд 14:29:46  385307
>>385304
>ты в черную дыру летишь.
>Для тебя то все ок
Ну такое. Были деньки и получше.
Аноним 28/02/18 Срд 16:18:41  385331
>>384183
>Правда ли, что дельфины такие умные, что могут мыслить? Мне кажется это пиздеж.
По крайней мере, зеркальный тест, свидетельствующий о наличии самосознания, дельфины проходят раньше, чем человеческие детеныши.
https://naked-science.ru/article/sci/delfiny-uznayut-sebya-v-zerkale-ranshe
Аноним 28/02/18 Срд 16:22:32  385334
>>385331
У сорок или ворон тоже самосознание есть.
Аноним 28/02/18 Срд 16:46:36  385337
>>385179
>>385192
>все что достигает горизонта событий этот самый горизонт пересекает уже на скорости света
Вы оба не совсем правы.
Горизонт событий, по определению – светоподобная граница. Если кто-то, падая в ЧД, в момент пересечения горизонта событий посветит фонариком назад, то свет навсегда зависнет на горизонте, не сможет ни улететь из дыры, ни упасть в нее.
Поэтому правильно сказать, что горизонт событий пролетит мимо падающего в черную дыру наблюдателя со скоростью света.
Говорить о скорости падающего в ЧД можно лишь условно, т.к. невозможно создать непрерывную цепочку причинно связанных систем отсчета во всей области внурти и снаружи черной дыры.
Аноним 28/02/18 Срд 16:52:14  385338
>>385334
Только у них ограничение на размер мозга. А так да, очень умные, только если они захотят стать ещё более умными, им придется стать нелетающими, а тут их быстро собаки с кошками схомячат.
Аноним 28/02/18 Срд 23:24:00  385385
>>385176
Я ХЗ.
https://ru.wikipedia.org/wiki/%D0%90%D0%BB%D0%BA%D0%B8%D0%BB%D1%8B
Аноним 28/02/18 Срд 23:39:23  385386
>>385179
Ну короч. смотри.
Представим что ты неразрушимая элементарная частица падающая в дыру. Приближаясь ты всё замедляешься и замедляешься. Сначала 4 м\с, потом 2м\с потом 1 и т.д. Но не прекращаешь падение. И вот спустя н времени для внешнего наблюдателя. ты приближаешься так близко, что твоя масса прибавляется к чд, и ГС как бы наезжает на тебя ввиду увлечения его радиуса.
С твоей точки зрения не изменяется ничего. Это лишь внешнему наблюдателю кажется что ты застыл на месте до скончания вселенной и должен был бы увидеть её конец. Но нет. Ты застывший во времени лишь отображение прошлого тебя, как ты и сказал чёрточка не зря существует. И ты уже непонятным образом путешествуешь, возможно ты попадёшь в Белую дыру, а возможно ты будешь лететь отдельно от текущего пространства-времени. А возможно ты как неразрушимая частица, разрушешься влившись в неотделимую 1 флуктуацию. Короче говоря неизвестно что под корочкой, но одно понятно что ты не зависнешь на её краю, а залетишь туда, что куда криповее чем если бы ты просто денатурировал в окрестностях.
Аноним 01/03/18 Чтв 10:06:27  385413
Возможно ли где нибудь существование газовых гигантов со спутником в пропорциях как Земля-Луна?
Аноним 01/03/18 Чтв 11:46:24  385422
>>385337
>Вы оба не совсем правы
Что значит не совсем, м, уеба. Покажи мне объект, обладающий массой покоя, достигающий скорости света. Или что, очередной борец со сказками усатого жида.
Аноним 01/03/18 Чтв 11:48:02  385423
>>385337
>свет навсегда зависнет на горизонте
Оу, пардон, многоуважаемый дон Эфир. Не сразу признал. Посыпаю голову пеплом и удаляюсь.
Аноним 01/03/18 Чтв 12:43:12  385432
>>385413
Маловероятно. Проблема в том, что столь массивное тело будет хавать весь газ вокруг и столь близкому соседу просто не из чего будет возникнуть.
Аноним 01/03/18 Чтв 12:57:16  385437
>>385413
>Возможно ли где нибудь существование газовых гигантов со спутником в пропорциях как Земля-Луна?
При штатном механизме формирования – нет. Кандидатами в спутники становятся планетезимали, конденсирующиеся из части протопланетного облака, аккрецируемого будущей планетой-гигантом. Чем больше масса потенциального спутника, тем быстрее он поглощает вещество из околопланетного диска и тем быстрее торзозится, падая в итоге на планету-гигант. Выживает лишь последнее поколение спутников, которым повезло сформироваться к тому моменту, когда запасы вещества в протопланетоном облаке иссякают, и для них исчезает торзмозящая среда. Поэтому существует жесткое ограничение на массу регулярных спутников, которое зависит от плотности начальной исходного газопылевого диска и для нашей системы составляет около 1/10000 массы центральной планеты.
Более масссывные спутники могут появляться либо в результате импактных событий, как Луна или Харон, либо в результате захвата, как Тритон. Понятно, что столкновениями с газовым гигантом спутник не сформируешь, остатеся только захват. Но для захвата будущий спутник либо сам уже должен быть компонентом двойной системы, чтоб второй партнер унес избыток энергии, либо столкуться с одной из меньших лун планеты-гиганта. Учитывая известные данные о формировании планетных систем, такие события будут крайне маловероятны.
Аноним 01/03/18 Чтв 14:24:55  385458
>>385179
Время останавливается только в самой точке сингулярности.
Аноним 01/03/18 Чтв 15:25:45  385470
>>385413
>>385432
Так-то находили сверхюпитер со спутником размером с уран.
Так что всё возможно. Конечно не на 380 000 км, а на порядки больше.
Да упадёт, но пару миллиардов лет продержаться может. Учитывая что у нас миллиарды звёзд, в нашей галактике, то может где есть в почти точности с пропорциями.
Аноним 01/03/18 Чтв 15:48:31  385478
278584162013717[...].png (327Кб, 860x482)
В случае апокалипсиса (зомби/метеоритного/снежного) как долго будет продолжать работать спутниковая связь и навигация?


пикрандом
Аноним 01/03/18 Чтв 15:51:48  385479
Суп спейсач, как ты наверное слышал, Пу недавно анонсировал первую в мире ракету на ядерном двигателе. Скажи окажет ли это какое-то влияние на космическую индустрию? Есть ли у технологии перспективы в космосе? Я знаю что США и СССР разрабатывали свои такие двигатели для космических ракет до 1970, но потом приоритеты изменились.
Аноним 01/03/18 Чтв 16:03:48  385484
>>385479
Не слышал, ссылку кинь. Это не за 2009 год новость?
Аноним 01/03/18 Чтв 16:16:28  385487
>>385484
https://meduza.io/slides/raketa-s-yadernym-dvigatelem-ognennyy-shar-i-lazery-putin-ustroil-prezentatsiyu-rossiyskogo-oruzhiya
Аноним 01/03/18 Чтв 16:18:48  385488
>>385479
Может быть что-то из этой степи https://ru.wikipedia.org/wiki/Supersonic_Low-Altitude_Missile а может быть двигатель в стиле Нервы, нихуя не понятно пока.
Аноним 01/03/18 Чтв 16:28:04  385490
>>385487
Выглядит как хуета, для суборбитальных сверхманевров с целью уклонения от про нужна хорошая тяга, а этого не добились как старые нерва и рд-0410, так и более поздний тимбервинд.
Вообще ядерные двигатели скорее всего не имеют будущего, только электрика и термояд это совсем не скоро.
Аноним 01/03/18 Чтв 16:43:53  385497
https://www.youtube.com/watch?v=tUPlltCA99c
Аноним 01/03/18 Чтв 16:45:25  385498
>>385497
Это не тупой вопрос, это тупой ответ.
Аноним 01/03/18 Чтв 19:03:22  385531
>>385478
Пару лет проработает.
Аноним 01/03/18 Чтв 19:20:20  385534
>>385497
Ой, удивил, блядь. Вооружение, вооружение, одни пушки на уме. На ЯГПВРД бы создать орбитальный самолёт, вот это круто будет
Аноним 01/03/18 Чтв 19:37:21  385535
>>385534
>одни пушки на уме
Напомни мне для чего использовались первые современные ракеты?
Аноним 01/03/18 Чтв 20:34:29  385555
>>385535
Да да, я знаю про редстоун и р-7, речь не об этом. На базе ракеты с гпврд создать самолёт не очень просто. Это разные конструкции, совершенно
Аноним 01/03/18 Чтв 20:44:42  385557
Существуют ли более красивые альтернативы оранжевой термоизоляции водородных баков?
Аноним 01/03/18 Чтв 20:49:02  385559
STS-1S81-30498S[...].jpg (817Кб, 3000x2528)
>>385557
Поверх этой оранжевой изоляции можно красить как душе заблагорассудится, но это лишний вес и всем похуй.
Аноним 01/03/18 Чтв 21:30:16  385560
>>385559
А я лет 15 назад в технике молодежи вроде видел огромные водородные шарики белого цвета с сегментированной структурой для кораблей дальнего космоса, это имеет реальную основу?
Аноним 01/03/18 Чтв 21:31:41  385563
>>385478
Недолго, у всех спутниковых систем огромная инфраструктура на земле
Аноним 01/03/18 Чтв 21:33:02  385564
>>385479
Так вот откуда у нас полстраны радиацией засрано
Аноним 01/03/18 Чтв 21:42:13  385566
>>385490
При чем тут нерва и рд-0410? Речь шла о таком, я тебе точно говорю. https://ru.wikipedia.org/wiki/%D0%93%D0%B8%D0%BF%D0%B5%D1%80%D0%B7%D0%B2%D1%83%D0%BA%D0%BE%D0%B2%D0%BE%D0%B9_%D0%BF%D1%80%D1%8F%D0%BC%D0%BE%D1%82%D0%BE%D1%87%D0%BD%D1%8B%D0%B9_%D0%B2%D0%BE%D0%B7%D0%B4%D1%83%D1%88%D0%BD%D0%BE-%D1%80%D0%B5%D0%B0%D0%BA%D1%82%D0%B8%D0%B2%D0%BD%D1%8B%D0%B9_%D0%B4%D0%B2%D0%B8%D0%B3%D0%B0%D1%82%D0%B5%D0%BB%D1%8C#%D0%AF%D0%B4%D0%B5%D1%80%D0%BD%D1%8B%D0%B5_%D0%B3%D0%B8%D0%BF%D0%B5%D1%80%D0%B7%D0%B2%D1%83%D0%BA%D0%BE%D0%B2%D1%8B%D0%B5_%D0%9F%D0%92%D0%A0%D0%94
Аноним 01/03/18 Чтв 21:48:10  385568
suborbital.jpeg (116Кб, 600x807)
Вопрос по черным дырам.
По определению, ниже горизонта событий скорость убегания для черной дыры превышает скорость света, поэтому ничего не может ее покинуть.
Однако, скорость убегания применима только к баллистическому полету. Предположим у нас есть корабль с мощным фотонным двигателем (УИ=c, тяга достаточна, что бы twr сразу под горизонтом событий был больше 1). Топливом будет пара материя/антиматерия, топливо составляет 95% массы корабля. По формуле Циолковского получим запас дельты ~3с. Да, это не позволит разогнаться до 3c, но можно, к примеру, будет разогнаться до ~0.45с, потом затормозить и так три раза с запасом. Корабль прямо под горизонтом событий, скорость убегания для него 1,01c. Корабль разгоняется до 0,95c, и летит по суборбитальной траектории вверх. В точке, где он затормозится до нуля, скорость убегания будет уже меньше с, и там он разгоняется повторно, опять до 0,95с. Теперь ему нет необходимости разгонятся выше световой скорости для покидания горизонта событий.
Вопрос, где тут ошибка?
Аноним 01/03/18 Чтв 22:16:29  385571
>>385568
>Корабль прямо под горизонтом событий
А первая космическая не будет там больше c?
>скорость убегания для него 1,01c. Корабль разгоняется до 0,95c, и летит по суборбитальной траектории вверх
Как он может выйти на суборбитальную траекторию, если он летит по орбите? Другое невозможно
А ну и ты не учел релятьявистскую массу
Аноним 01/03/18 Чтв 22:24:21  385572
>>385566
Хуй знает, я вывод сделал на основе заявлений сомнительной достоверности о орбитальных маневрах, а там воздуха нет.
Аноним 01/03/18 Чтв 22:33:42  385573
>>385572
Там не было речи про орбитальные манёвры. Про просто манёвры было. Да и ярд на мбр это очень тупая идея
Аноним 02/03/18 Птн 00:13:36  385584
Как проще представить 'ось' w четырех мерного пространства? Если это возможно. Я понимаю, что сечение куба это квадрат, сечение квадрата это отрезок,а отрезка уже точка и понятно, что куб это сечение тессеракта (?) В теории понятно, но хоть как-то визуализировать не могу
Аноним 02/03/18 Птн 00:29:00  385585
bc.gif (1342Кб, 500x500)
>>385568
Ты СОВЕРШЕННО не понимаешь суть Чёрной дыры. Чёрная дыра это не нейтронная звезда, "о прикиньте, плотность на порядок выше, чем у атомных ядер". Чёрная дыра это не остывший чёрный карлик. Чёрная дыра – это не звезда тёмной энергии, гравастар или RP3-геон. Черная дыра – это такая область пространства-времени, где оно само безудержно падает в сингулярность – меньше любого предела измерений, не имеющую волос, безразличную к любым стремлениям разумной жизни, с которых все началось, и которые будущем для всего сущего на самом деле и являются.

Жалкие людишки запустили корабль, топливом для которого служит пара материя/антиматерия, а он скатывается в сингулярность. Топливо составляет 95% массы корабля, а он скатывается в сингулярность. По формуле Циолковского запас дельты ~3с, а он скатывается в сингулярность вместе с самой тканью пространства-времени, которая ниже горизонта событий падает к центру со скоростью выше световой.
Поглощены несколько шаровых скоплений с десятками тысяч пригодных для жизни планет — она поглощает ещё. Сверхэддингтоновская скорость аккреции — она поглощает и просит ещё. Полярные джеты стерилизовали несколько заселенных планет — она поглощает дальше. Плазма, нейтральный водород, газопылевые облака, целые карликовые галактики — она поглощает всё. Другие чёрные дыры — она поглощает их тоже. Она бездушно поглотит всё, что угодно, ее поведение основано на строгих уравнениях ОТО, бесцельное накопление максимально возможной энтропии — её стихия, она — истинное лицо конечной эры Вселенной.
02/03/18 Птн 00:34:06  385586
40153877480855d[...].jpg (112Кб, 500x436)
>>385584
>визуализировать
можно только проекции тессеракта
Аноним 02/03/18 Птн 02:10:17  385595
Опасность.webm (9979Кб, 1280x720, 00:01:37)
>>385568
Короч. смотри, приближаясь к скажем земле ты увидишь, как земля увеличивает свою видимую величину. Сначала 1 градус, затем 5, и под конец когда ты приблизишся к асфальту 180+- градусов. С чёрной дырой совсем не так. Всё начинается обыденно. Сначала 1 градус, затем 5. Когда ты достигаешь нескольких метров над ГС, видимый угол становится 180 градусов. 180 градусов абсолютной темноты. Если приблизится ещё ближе, угловой радиус будет уже 190 градусов, затем 270, 345. Когда же ты приближаешься уже в притык, градусов уже 359,999999. Угадай что произойдёт когда ты его пересечёшь? Куда ты будешь лететь?
Аноним 02/03/18 Птн 02:18:16  385598
000007-3ot5hvyl[...].jpg (144Кб, 1154x325)
>>385386
Спасибо тебе анон, это немного помогло )
Буду стремится дополнить свои знания и обновить картинку, как опять замкнёт вкачусь с очередным тупым вопросом
Аноним 02/03/18 Птн 05:36:52  385610
>>385586
То есть никто не знает как он толком выглядит? Но лишь примерно понимают его "законы"?
Аноним 02/03/18 Птн 07:16:18  385613
>>385595
А посоны из http://jila.colorado.edu/~ajsh/insidebh/schw.html пишут, что подобные эффекты типа сжатия поля зрения в точку возникают исключительно по причине движения с ускорением, при свободном падении в черную дыру нихуя подобного не произойдет. Вплоть до самой встречи с сингулярностью черная дыра не займет больше 180° неба, и при пересечении горизонта событий не будет вообще никаких заметных эффектов.

>Engulfed in blackness? NO! It is a common misconception that if you fall inside the horizon of a black hole you will be engulfed in blackness.
>More specifically, the story is that as you fall towards the horizon, the image of the sky above concentrates into a smaller and smaller circular patch, which disappears altogether as you pass through the horizon. The misconception arises because if you lower yourself very slowly towards the horizon, firing your rockets like crazy just to stay put, then indeed your view of the outside universe will be concentrated into a small, bright circle above you. Click on the button to see what it looks like if you lower yourself slowly to the horizon.
>Physically, this happens because you are swimming like crazy through the inrushing flow of space (see Waterfall), and relativistic beaming concentrates and brightens the scene ahead of (above) you. See 4D Perspective for a tutorial on relativistic beaming.

Википидоры в статье http://en.wikipedia.org/wiki/Event_horizon тоже согласны:

>Observers crossing a black hole event horizon can calculate the moment they have crossed it, but will not actually see or feel anything special happen at that moment. In terms of visual appearance, observers who fall into the hole perceive the black region constituting the horizon as lying at some apparent distance below them, and never experience crossing this visual horizon. Other objects that had entered the horizon along the same radial path but at an earlier time would appear below the observer but still above the visual position of the horizon, and if they had fallen in recently enough the observer could exchange messages with them before either one was destroyed by the gravitational singularity. Increasing tidal forces (and eventual impact with the hole's singularity) are the only locally noticeable effects.
Аноним 02/03/18 Птн 09:24:24  385619
>>383656 (OP)
Нахуя нужна вся современная космонавтика, если Россия может создавать гиперзвуковые самолеты со скорость в 20 махов? Чутка махов добавить и все одноступенчатая космическая система готова. А там и до установки на нее варпдвигателя недалеко.
Аноним 02/03/18 Птн 09:43:26  385620
Почему все планеты солнечной системы вращаются в одной плоскости?
Аноним 02/03/18 Птн 10:32:26  385621
>>385619
>если
Аноним 02/03/18 Птн 10:47:52  385623
>>385619
Когда сможет, тогда и поговорим.
Аноним 02/03/18 Птн 10:50:29  385624
>>385620
Образовались из одного диска.
Аноним 02/03/18 Птн 10:58:35  385627
Это не тупой вопрос, скорее тупой баттхерт. Сука, почему современные АМС такое убогое гавно, спаянное студентами в шараге на коленке за овердохуя бабла? И я не о фобосвгрунтах, скиапарелли и чандраянах, с ними-то как раз-таки все понятно. Вот например предлагается дрочить на Куриосити с его скайкрейном, MMRTG и лазерным спектрометром. Но и у него был дичайший перерасход средств, ПЕРЕНОС, дырявые колеса и факапы с механизмом отбора проб грунта. И так практически в любой сколько-нибудь масштабной и технологически нетривиальной миссии - оверпрайснутость и/или дурацкие обосрамсы на ровном месте, от которых страдает научная ценность миссии и репутация околокосмических социальных институтов. Недавно умучили WFIRST из-за поебенек с Веббом. Почему нельзя делать все по красоте, чтобы не было такой хуйни? Изменится ли ситуация с приходом частников в космос?
Аноним 02/03/18 Птн 11:20:42  385632
>>385627
Так частники и так выступают в роли подрядчиков. Ты думаешь что все эти роверы-хуеверы и амсы наса пилит в соло на своих заводах и кб, ага?
Аноним 02/03/18 Птн 11:52:05  385635
>>385632
Да не, я в курсе про JHU/APL, Northrop Grumman и прочие конторки. Сорян за неточную формулировку. Просто IRL, как на мой тупой взгляд, они де факто и являются такими КБ с отсутствием здоровой конкуренции ололо-рыночка. Смогут ли новые частники, типа Маска, Безоса и Бигелоу здесь что-то зарешать, или ограничатся средствами доставки?

Только что прочел в соседнем треде про очередной перенос Вебба с неясными перспективами. Бля.
Аноним 02/03/18 Птн 12:07:33  385637
>>385635
Муски являются по сути такими же насами, и так же отовариваются у подрядчиков.
Аноним 02/03/18 Птн 12:07:54  385638
>>385627
Потому что это очень сложные устройства, которые должны работать в хуй пойми каких условиях без возможности починки/исправления на месте. Это требует кучи проверок и использования сомнительных решений при разработке.
Собственно, в этих решениях вся проблема - отправишь обычную радиомашинку на Марс - она сломается из-за какой-нибудь марсианской пыли или застрянет на марсианском камне. Поэтому нужно предусматривать все ситуации и принимать меры по их недопущению, отсюда переусложнение конструкции, повышение стоимости и сроков, но и увеличение шансов на успешность миссии в целом.
Подразумеваемые тобой частники типа муска и лысого вообще наукой и исследованиями не занимаются кроме непосредственно разработки своих ракет, в настоящее время они просто занимаются грузоперевозкой, скажем так.
А в остальном АМСы делаются как раз кучей частников
Аноним 02/03/18 Птн 12:10:12  385639
>>385627
>Сука, почему современные АМС такое убогое гавно
Оно не убогое говно. Оно передовое. Нужно склепать хуйню, которая бы пережила несколько месяцев, или лет полета в вакууме при радиации, а потом еще работало в каких-то фантастических условиях, которые на Земле даже смоделировать трудно. При том вся эта ебала упирается в ограничение по массе.
То есть так-то можно было бы все это засунуть в корпус какого-нибудь танка, все манипуляторы имели бы толщину с ногу слона, провода сечением в палец, а микросхемы с кулак. Но это невозможно.
Все современные АМС - это невероятно фантастическая хуета, по сравнению с убогими хуйнями 70-80-х.
Аноним 02/03/18 Птн 12:10:23  385640
>>385635
Не, там конкуренция между разными фирмами за контракт на изготовление той или иной хуйни для аппарата.
И да, частники нихуя не панацея, это всего лишь свежий взгляд, не более. В будущем они или остановятся, или станут такими же неповоротливыми гигантами как наса.
Аноним 02/03/18 Птн 12:40:29  385647
Bt-ph-294-2.jpg (17Кб, 500x419)
>>385639
>по сравнению с убогими хуйнями 70-80-х
А вот сейчас было обидно.
Аноним 02/03/18 Птн 13:01:17  385650
>>385610
Нет, математики, что работают с этой хуйней, вполне себе реально представляют эти объекты.
Аноним 02/03/18 Птн 13:06:28  385651
>>385620
Потому же, почему галактики плоские, диски акреции черных дыр плоские.
Вот взять вращающийся шар из хуйни.
Материя будет притягиваться друг к другу у полюсов.
А возле экватора не будет из-за центробежной силы, ибо вся эта хуйня вращается.

Так как всегда из вращающейся кучи хуйни возникает диск из хуйни, то уже из этого диска и формируются планеты
Аноним 02/03/18 Птн 15:18:47  385663
galileoantennah[...].gif (7Кб, 150x198)
BBUzOxgCYAA8n30.jpg (86Кб, 600x453)
Curiosity-1641M[...].jpg (114Кб, 780x650)
>>385638
>>385639
Ну лан, но почему, если они такие охуенные, то так часто проябываются? Единственное, живучесть у них стала поболе.
Аноним 02/03/18 Птн 17:26:35  385682
>>385663
>то так часто проябываются?
Что там у тебя часто проёбывается?
Опортьюнити, который накатал 45 км за 14 лет по Марсу?
Розетта, которая 12 лет мотылялась по СС?
MSL, уоторая уже работает в три раза дольше, чем было запланировано?
Или может Кассини, пролетав 20 лет тебе не угодил?
Аноним 02/03/18 Птн 17:53:47  385684
>>385663
Не могу вспомнить ни одного критического проеба насы в этом веке. Джуны, осирисы, давны, кеплеры, свифты, вайсы всё живое до сих пор.
Аноним 02/03/18 Птн 17:58:41  385686
>>385682
Мыльная параша Junocam
Аноним 02/03/18 Птн 18:00:37  385687
>>385686
Это вообще бичевский аппарат и камеру приклеили изолентой в последний момент.
Аноним 02/03/18 Птн 18:26:18  385690
>>383656 (OP)
слева хаббл?
Аноним 02/03/18 Птн 19:36:14  385698
>>385686
>>385687
Вы оба мудаки, и даже лень объяснять, почему.
Аноним 02/03/18 Птн 20:06:34  385705
>>385682
В зенки долбишься? Про живучесть я написал, только живучесть эта - живучесть инвалидов, ковыляющих на костылях. У Опортьюнити не работают самые важные приборы. У Розетты случился имбецильный провал заякоривания посадочного зонда (из-за чего тот не выполнил самые важные эксперименты), а ее саму уебали о ядро кометы под смехотворным предлогом. MSL ковыляет на проткнутых колесах с едва работающим буром. С Кассини все более-менее было норм, но вот, например, с Гюйгенсом сначала пришлось вносить изменения в программу полета из-за того, что не был учтен допплеровский сдвиг частоты при разных скоростях его и Кассини, но при посадке все равно половина фоток и данные анемометра были проебаны, так как кто-то забыл включить (sic!) соответствующий приемник на орбитере. И это только самые жидкие обсрамцы.
Аноним 02/03/18 Птн 20:07:07  385706
>>385690
Эддингтон же.
Аноним 02/03/18 Птн 21:20:24  385722
>>385651
То есть в принципе возможно пустить какой нибудь объект в другой плоскости вокруг солнца?
Аноним 02/03/18 Птн 21:53:57  385730
>>385722
Если хватит энергии для этого.
Например астероиды сталкиваются где-то в жопе системы а там для смены орбиты нужно намного меньше энергии и потом летают как попало. Или залетные тела, у них вообще никаких ограничений на орбиту нет.
Ну или просто включить двигатели и на ретроград.
Аноним 02/03/18 Птн 22:10:55  385733
>>385278
>Нахуя мы на миллиард лет назад смотрим?
не хочешь - не смотри
Аноним 02/03/18 Птн 22:25:09  385743
>>385497
Там кое что поинтереснее:
>Также президент заявил о проведенных в стране испытаниях «Авангарда» — гиперзвукового ракетного комплекса с планирующим крылатым блоком, который «отличается способностью совершать полеты в плотных слоях атмосферы на межконтинентальную дальность, на гиперзвуковой скорости, превышающей число Маха более чем в 20 раз». По словам Путина, «он идет к цели, как метеорит, как огненный шар, температура на поверхности изделия — 1600-2000 градусов по Цельсию», а «крылатый блок при этом надежно управляется». Подобные характеристики комплексу, отмечает глава государства, обеспечило использование композитных материалов.
Это возможно? Они просто засунули в союз ядерную боеголовку? Нахуя, главное такого рода оружие?
Аноним 02/03/18 Птн 23:03:55  385758
>>385743
Ты не можешь отличить предвыборный звездежь от здравого смысла?
Аноним 02/03/18 Птн 23:10:22  385761
>>385758
>>385743

Тут как раз ничо особого нет. re-entering maneurable warhead и всё такое. Боеголовка теперь при движении в атмосфере не просто падает, а имеет некоторое значительное аэродинамическое качество, и рулит туда-сюда, чтоб тяжелее в неё было попасть.
Аноним 03/03/18 Суб 10:02:22  385828
>>385761
Так я и спрашиваю, они засунули в союз боеголовку?
Аноним 03/03/18 Суб 10:06:53  385829
>>385828
В очко тебе засунули боеголовку... Если ты понимаешь о чем я ;)
Аноним 03/03/18 Суб 12:22:28  385873
>>385585
Но ведь, излучения Хокинга...
Аноним 03/03/18 Суб 13:01:53  385878
Когда перестанем кипятить воду во всяких йобах?
Аноним 03/03/18 Суб 13:03:39  385879
>>385878
Когда ты, анон, придумаешь не менее эффективный способ получать электричество.
Аноним 03/03/18 Суб 13:25:56  385889
image.png (178Кб, 800x209)
>>385879
Типа такого?
Аноним 03/03/18 Суб 13:27:28  385890
image.png (213Кб, 598x337)
>>385889
Это что такое-то?
Аноним 03/03/18 Суб 13:47:05  385898
>>385890
Прямая конверсия кинетической заряженных частиц термоядерного синтеза в электричество.
Аноним 03/03/18 Суб 13:48:23  385899
>>385898
Недурно.
Осталось всего-то освоить термояд с положительным выхлопом.
Аноним 03/03/18 Суб 13:56:05  385902
>>385828
К падающему боевому блоку прикрутили крылышки, в результате чего он научился рулить в атмосфере.

Про 20 мах - чистый популизм уровня /b.
Во-первых, скорость звука на больших высотах в разряженной атмосфере гораздо ниже, чем у поверхности.
Итого, 330 м/с у воды - это 1 мах, а 330 м/с на высоте 40 км - это уже 2-3 маха.

Так как боевой блок влетает в атмосферу на суборбитальной скорости, то ясен хуй у него будет дохуя махов.

Впрочем, как и любых других блоков.
Аноним 03/03/18 Суб 13:56:16  385903
>>385898
А можешь по-подробнее описать принцип?
Аноним 03/03/18 Суб 14:04:46  385905
image.png (1340Кб, 1356x1017)
image.png (237Кб, 600x283)
>>385903
https://en.wikipedia.org/wiki/Direct_energy_conversion
>>385899
Ну вообще подобное можно проворачивать на специфических ядерных реакторах, типа пылевой плазмы, это поближе термояда.
Аноним 03/03/18 Суб 15:02:30  385916
14905584615500.jpg (2277Кб, 1500x1500)
Допустим мы узнали, что из другой звездной системы в 2000 световых лет от нас в сторону Солнца отправлен снаряд. Траектория снаряда настроена по эфемеридам или как-то так, предсказывает позицию Солнца в рукаве галактики. Можно ли сманеврировать Солнечной системой, замедлить или ускорить движение Солнца в рукаве галактики за счет гравитационного маневра пролетающей через солнечную системы планеты изгоя? То есть уклониться от возможного "снаряда" пришельцев, который не имел бы коррекции. Как еще можно задурить систему навигации снаряда? Если спрятать Солнце за мощной вспышкой сверхновой, изменить спектр Солнца, закрыть его какой-то конструкцией наподобие тени?
Аноним 03/03/18 Суб 15:05:34  385917
>>385916
послать свою ракету чтоб врезалась во вражескую раньше чем система долетит до точки назначения
Аноним 03/03/18 Суб 15:05:37  385918
>>385916
Можно просто каким-либо образом повлиять на снаряд.
Аноним 03/03/18 Суб 15:12:33  385920
>>385916
> который не имел бы коррекции. Как еще можно задурить систему навигации снаряда?
Ебать ты дурень, какая навигация в неуправляемом снаряде? Хуевые вопросы у тебя короче. На 2000 св лет неуправляемым хуй попадешь при любом уровне технологий кстати.
Аноним 03/03/18 Суб 15:14:56  385921
999px-Near-star[...].png (110Кб, 999x800)
>>385917
>>385918
Может так статься, что снаряд летит с околосветовой скоростью. Как вы его тогда будете сбивать? Допустим мимо вас один раз пролетел такой снаряд, он пролетел мимо именно из-за того, что ваша система замедлилась из-за гравитационного маневра с коричневым карликом. Когда этот снаряд послали - они этого не предвидели, поэтому вы стали свидетелем как в 10 световых годах от вас пролетела верная смерть в виде чего угодно - допустим карликовой голой сингулярности. И вы понимаете, что это говно должно было по идее уничтожить вашу звезду.

Лучше заранее сделать движения своей звездной системы непредсказуемыми. Так чтобы нельзя было составить никаких карт вашей звездной империи, чтобы никто никогда не мог сказать где ваш homeworld. То есть "они" может и будут знать, но их информация всегда будет устаревшей. Может к моменту, когда их снаряда долетят до вашего хоумворлда вы эвакуируете все планеты из звездной системы. Полнейшая непредсказуемость условие выживаемости в условиях, когда вам противостоят разумные противники.

>>385920
Хочешь сказать, что движения звезд непредсказуемы на такие сроки и дистанции? Откуда мы тогда знаем пикрелейтед позиции и дистанции?
Аноним 03/03/18 Суб 15:28:27  385924
>>385921
Ну тогда посветим в него лазером скорость света, он чуток изменит направление и промажет, все. Может даже взорвется, если хорошо посветим.
>Хочешь сказать, что движения звезд непредсказуемы на такие сроки и дистанции? Откуда мы тогда знаем пикрелейтед позиции и дистанции?
Примерно. Во первых у них недостаточная для попадания точность, во вторых на снаряд будет действовать любое встречное говно, начиная от огромных объектов которые можно учесть, заканчивая полностью случайной так называемой космической погодой.
Это примерно как с обычными артиллерийскими снарядами - у них разброс километры, а управляемой ракетой можно попасть в круг 5х5 с расстояния в десять раз больше.

Воообще с этим дерьмом в сайфач пиздуй, технологии перемещения планет это пиздец отрыв от реальности и сейчас обсуждать это даже хуже чем тыкание пальцем в небо.
Аноним 03/03/18 Суб 15:37:52  385927
>>383656 (OP)
Кто из частников первый запустил ПН и человека в космос?
Аноним 03/03/18 Суб 15:38:23  385929
>>385927
Илон. Человека - никто.
Аноним 03/03/18 Суб 16:49:39  385953
>>385929
Ты тупой или тупой?
Аноним 03/03/18 Суб 16:51:00  385954
>>385924
>Это примерно как с обычными артиллерийскими снарядами - у них разброс километры
Хуёвые у вас там артиллеристы.
Аноним 03/03/18 Суб 16:59:35  385955
>>385916
Очевидно перехватить снаряд проще, дешевле и надежнее всего.
Аноним 03/03/18 Суб 17:02:20  385956
>>385921
> Может так статься, что снаряд летит с околосветовой скоростью
Значит крайне хуево маневрирует и его траектория предсказывается с очень большой точностью.

Начать можно с того, что поставить на пути снаряда камушек.
При подлете облучать с лазеров
Аноним 03/03/18 Суб 17:05:39  385958
>>385956
>снаряд летит с околосветовой скоростью
Вопрос пока только один: как его заметить до того, как он пересечёт орбиту Плутона?
Аноним 03/03/18 Суб 17:06:31  385960
>>385955>>385956

Ну вот летит там "снаряд" со скоростью 95% от скорости света, у тебя время обнаружения его почти когда он уже к тебе подлетел. В качестве "снаряда" допустим гигантский черный корабль, который распадается на множество мелких кораблей, который бомбардируют все планеты черными треугольниками с антиматерией. Я ебу что у них там в моде? Может они просто пошлют в нашу систему гигантский кусок плутониево-калифорниевой руды, который упав на Солнце вызовет там магнитную бурю эпических масштабов.
Аноним 03/03/18 Суб 17:08:46  385961
>>385953
Доказывай обратное или продолжай кидаться ничего не значащими оскорбленияи.
Аноним 03/03/18 Суб 17:12:41  385962
>>385958
Ну, 0.5c тоже околосветовая.
А если 0.9999, тогда проблемы, да
Аноним 03/03/18 Суб 17:15:06  385963
>>385960
Тогда хуярить лазерами.
Аноним 03/03/18 Суб 17:15:15  385964
Проходят ли в солнце химические процессы, вроде образования воды?
И влияют ли они на светимость и прочее хоть в какой-то степени?
Аноним 03/03/18 Суб 17:16:02  385965
>>385964
Нет, слишком жарковато чтоб химические связи образовывались.
Аноним 03/03/18 Суб 17:26:59  385967
Почему дефект масс не работает в обратную сторону при синтезе гелиев и прочих углеродов? Чому в них нинужно вливать энергию, чтобы они склеивались, а они наоборот выделяют ее?
Аноним 03/03/18 Суб 17:30:36  385968
>>385967
Потому что ты не понимаешь сути деффекта масс
Аноним 03/03/18 Суб 17:31:14  385969
>>385968
Да как же так! Дефекта
Аноним 03/03/18 Суб 17:31:55  385970
>>385961
Перестань троллить тупостью и ползи обратно в б, неуч
Аноним 03/03/18 Суб 17:34:28  385971
>>385902
>скорость звука на больших высотах в разряженной атмосфере гораздо ниже, чем у поверхности
Ниже, да не настолько.
>0 м 340,3 м/с
>40000 м 317,2 м/с
https://ru.wikipedia.org/wiki/Стандартная_атмосфера

Так что 20 Махов все равно будет 6,3 км/с даже на высоте 40 км.
Поэтому приведенное заявление – пиздне соответствует действительности, а Путин – хуйллжец.
Аноним 03/03/18 Суб 17:37:59  385973
>>385970
Сам перестань жыром истекать.
Первый коммерческий запуск был Дрэгон в мае 2012 к МКС.
Пилотируемых коммерческих еще ни одного не было ни разу.
Аноним 03/03/18 Суб 17:44:39  385978
>>385968
Дефект масс это просто энергия, как в батарейке. Почему одни атомы выделяют ее при склейке, а другие при расклейке?
Аноним 03/03/18 Суб 17:46:21  385979
>>385973
У тебя частники это только муск и лысый, да?
Аноним 03/03/18 Суб 17:47:31  385980
>>385979
Да. Остальные хоть и типа отдельные конторы работали исключительно по заказу НАСА.
Аноним 03/03/18 Суб 17:47:36  385981
>>385978
Потому что в атоме есть еще более компактные структуры со своими энергетическими уровнями, которые высвобождают эту энергию, а иногда поглощают.
Аноним 03/03/18 Суб 17:49:53  385982
>>385980
А типа муск не по приказу от насы пускает фалконы в рамках crs?
Аноним 03/03/18 Суб 17:51:32  385983
>>385982
Он не запиливал дрэгон по заказу, он изначально запиливал космический аппарат и в итоге его арендует НАСА для снабжения МКС.
Аноним 03/03/18 Суб 17:57:28  385984
>>385983
Хуйню не неси, муск подписался под crs когда еще фалкон 9 и драконы не летали.
Аноним 03/03/18 Суб 17:58:22  385985
image.png (710Кб, 1000x654)
>>383656 (OP)
Черная дыра похожа на пятак хряка.
Аноним 03/03/18 Суб 17:58:42  385986
>>385984
>SpaceX began developing the Dragon spacecraft in late 2004, making a public announcement in 2006 with a plan of entering service in 2009.[20] Also in 2006, SpaceX won a contract to use the Dragon spacecraft for commercially supplied resupply services to the International Space Station for the American federal space agency, NASA.
Разработку начали за два года до подписания контракта с НАСА.
Аноним 03/03/18 Суб 18:01:46  385987
>>385960
>летит там "снаряд" со скоростью 95% от скорости света
>пошлют в нашу систему гигантский кусок плутониево-калифорниевой руды
Лол, при такой скорости можно послать абсолютно что угодно, хоть гигантский кусок дерьма птицы Рух, эффект будет одинаковый. Когда в старших классах будешь проходить релятивисткую формулу полной энергии, то узнаешь, что кинетическая энергия тела, движущегося со скоростью 0,95с примерно в 200 раз выше той, что выделяется при цепной реакции распада калифорния.
Только бомбардировать такими снарядами нужно не Сольнце, а планеты – при достаточной продолжительности воздействия экстерминатус гарантирован.
Аноним 03/03/18 Суб 18:03:16  385988
>>385965
Так не в центре же. 6000 слишком много?
Аноним 03/03/18 Суб 18:04:23  385990
>>385988
Много. Большей частью даже электроны уже не держатся вокруг ядер, какие еще химические реакции?
Аноним 03/03/18 Суб 18:05:38  385991
>>385986
Разработку начали раньше, а запустили по контракту. Не было бы контракта, не было бы и запуска.
Аноним 03/03/18 Суб 18:07:54  385992
>>385991
А кто еще делал КА без заказа НАСА?
Аноним 03/03/18 Суб 18:09:43  385993
>>385986
А это, в те времена были другие объекты для космического снабжения или муск планировал договорится с роскосым вместо насы?
Аноним 03/03/18 Суб 18:10:33  385995
>>385993
Он же собирался ред дрэгон сделать и съебать со сраной земляшки.
Аноним 03/03/18 Суб 18:14:25  385996
>>385995
Это никак не меняет того факта, что у драгона была и есть всего одна задача - снабжение мкс через договоры с наса.
Я вообще твоей позиции не понимаю, у тебя какая-то своя система деления на частник / не частник и ты усираешься за нее чисто из принципа походу.
Аноним 03/03/18 Суб 18:20:56  385999
>>385996
Ну какие боинг с юлой частнки? Они только технически не принадлежат государству а на деле скачут под дудочку сената, все что скажут - сделают. И наоборот - сенат под их дудочку лобби скачет.
Безос и Маск - настоящие частники без влияния на наих и от них.
Аноним 03/03/18 Суб 18:36:19  386002
>>385978
Потому что на график посмотри, дурень
Аноним 03/03/18 Суб 18:41:08  386005
>>385999
Т.е. как только частник разрастается он перестает быть частником или как?
Аноним 03/03/18 Суб 18:41:28  386006
>>385999
Т.е. как только частник разрастается он перестает быть частником или как?
Аноним 03/03/18 Суб 18:48:59  386007
>>385999
>боинг с юлой
Кстати, я об этом раньше как-то не думал. Это ж коммерческие компании, ёпта.
Это что ж получается, Маск не первый космический частник, а всего-навсего первый космический ИП?
Аноним 03/03/18 Суб 18:50:44  386008
>>386006
А то! Через 20 лет здесь будут орать что спейсикс говно и полильщики, а вот <космическая компания будущего> дают всем пососать.
Аноним 03/03/18 Суб 18:51:29  386009
>>386007
Маск же не ИП.
Аноним 03/03/18 Суб 19:00:12  386012
>>386007
Маск просто основал свою ракетную шаражку с нуля, в этом все отличие.
Аноним 03/03/18 Суб 19:47:47  386021
>>385992
Тот кто хочет этот заказ. И заранее готовится, как какой-нибудь стадион который сначала строят, а потом выбираю для соревнований, а не выбранный недостраивают и он приходит в упадок.
Аноним 03/03/18 Суб 19:52:59  386023
>>386012
С нуля? т.е. запускал сначала сорбитовые ракетки, сам разрабатывал чертежи принципиально новых ракет, повторяя шаги фонбрауна и королёва? Или он всё же брал чертежи наса как основу? И естественно он начинал, как малый бизнес, божественные фотки с гоупро всего за 9,99$ ?
Аноним 03/03/18 Суб 20:37:52  386029
А все мы из остатков одной звезды сделаны или есть вероятность >10%, что некоторые люди из других? сойдёт за варик к тёлке подкатить?
Аноним 03/03/18 Суб 20:44:48  386031
Что окружающие думают про ваше астро-хобби?
Аноним 03/03/18 Суб 20:53:45  386032
>>386029
>сойдёт за варик к тёлке подкатить?
Не понял в чём проблема, ты опасаешься, что она проверит, из какой звезды твои атомы, а из какой её?
>>386031
Никому из окружающих не следует знать, что я дрочу не на сисятых тёлок или писюнястых трапов, а на турбонасосы ЖРД.
Аноним 03/03/18 Суб 20:58:17  386034
>>386032
Я просто как-то раз сказал... Батя посмотрел на меня как на долбоеба. Он считает это обсолютно бесполезным и лучше б я из универа не вылетел.
Аноним 03/03/18 Суб 21:04:25  386035
>>386034
Всяко лучше, чем хуита которой он увлекается.
Аноним 03/03/18 Суб 21:07:01  386036
>>386035
Ну.... Он Трейдер на бирже с зп в 900ксекунду месяц.
Аноним 03/03/18 Суб 21:11:15  386038
>>386034
Ну я б не сказал, что твой батя неправ. Большинство хобби почти полностью бесполезны - кроме того, что они приносят удовольствие конкретно тебе - и это норма. Если твоё хобби не несёт очевидных профитов, прагматичный батя смотрит на тебя как на долбоёба.
Аноним 03/03/18 Суб 21:12:29  386039
>>386032
Нет, я вообще подкатывать не собирался, а хотел на первый вопрос ответ узнать.
Аноним 03/03/18 Суб 21:21:49  386041
>>386036
Раньше это называлось спекулянт. Не зря в прошлом их гнали и насмехались над ними. По сравнению с этим даже коллекционирование елдаков в своей жопе куда более достойное занятие. Что уж говорить о таком высокодуховном увлечении как космос.
Твой батя одна из причин нестабильности мировой экономики.
Аноним 03/03/18 Суб 21:36:33  386043
Какую футболку купить: с ULA, Spacex или NASA?
Аноним 03/03/18 Суб 21:39:07  386044
>>386043
POCKOCMOC, очевидно же.
Аноним 03/03/18 Суб 21:45:24  386045
>>386043
Очевидная NASA.
Аноним 03/03/18 Суб 21:47:21  386046
image.png (88Кб, 1040x1024)
>>386043
Лучшее агентство, не запустившее ни одной ракеты, хотя имевшее их в разработке.
Аноним 03/03/18 Суб 21:48:56  386047
>>386039
Не, все в нашей системе образовалось из аккреционного диска Солнца.
Аноним 03/03/18 Суб 21:50:43  386048
image.png (160Кб, 409x409)
>>386043
ГЕПТИЛ СИЛА
МЕТАН МОГИЛА
Аноним 03/03/18 Суб 21:51:56  386049
изображение.png (130Кб, 800x369)
>>386043
Аноним 03/03/18 Суб 21:54:29  386050
000012-hwhzgfmd[...].jpg (486Кб, 800x568)
>>386043
Купи футболку с принтом земли-чан, только не новомодную из а и форча, а нашу каноничную.
А ещё лучше надпись снизу, "горжусь спейсачём!"
А наса это всего лишь организация космических полётов америки. Можешь конечно и её если фанат америки. Спесх это субподрядчик наса, а юла вообще министерство обороны сша.
То же самый эфект будет если ты будешь носить майку с ракетоносителем "булава" или майкой госкорпорации роскосмос. Выхлоп тот же толко страна другая. И если уж выбирать, то тогда уж японское агенство. Мы ведь на борде для аниме-девочек. Но я бы лучше космос нарисовал или кербалов, нежели какую-то организацию человеков.
Аноним 03/03/18 Суб 21:57:36  386051
>>386046
А откуда у них тогда ядерные заряды?
У них что они бомболючные что ли?
Аноним 03/03/18 Суб 22:01:11  386052
image.png (67Кб, 1200x746)
>>386050
Япошки кстати заебок вариант, неплохой символ плюс очень успешная для своего бюджета космическая программа.
Аноним 03/03/18 Суб 22:04:42  386053
>>386047
А солнце из одной мамко-звезды или много разных звезд в облако напердели чтоб образовалось солнце с СС?
Аноним 03/03/18 Суб 22:05:31  386054
svtfoe delet th[...].gif (27Кб, 276x337)
>>386050
>ракетоносителем "булава"

Аноним 03/03/18 Суб 22:06:15  386055
>>386051
Космических ракет нету. Баллистические запилили.
Вон, у французов так вовсе нету МБР наземного базирования, к слову, а уже какой Ариан подряд ебашут.
Аноним 03/03/18 Суб 22:06:25  386056
>>386053
Хуй знает
Аноним 03/03/18 Суб 22:29:18  386063
>>386055
Ты чо охуел, у них целый один успешный запуск на орбиту был ~45 лет назад.
Аноним 03/03/18 Суб 22:31:39  386065
>>386032
>я дрочу не на сисятых тёлок или писюнястых трапов, а на турбонасосы ЖРД
Сказал "а" - кидай картинки
Аноним 03/03/18 Суб 22:57:10  386070
>>386054
Ну, в принципе, разводящий блок вполне себе ракета сам по себе. Нестандартной формы.
И "Булава" его несёт.
Так что да, "Булава" — ракета-ракетоноситель!
Аноним 03/03/18 Суб 23:02:14  386075
abutthurt.jpg (294Кб, 580x3578)
>>386070
>ракета-ракетоноситель
Аноним 03/03/18 Суб 23:04:50  386076
Когда будет миссия по возврату грунта с Венеры?
Аноним 03/03/18 Суб 23:08:55  386080
>>386076
В 2280.
Аноним 03/03/18 Суб 23:09:38  386081
1398061911996.png (757Кб, 609x600)
>>386070
Удоли.

>>386065
Хотел старый тредшот скинуть где анон хвалил попец Протона, но не нашел.
Аноним 03/03/18 Суб 23:10:06  386082
>>386076
>по возврату грунта с Венеры
Зайдём с другой стороны: кто этот грунт туда забросил и зачем?
Аноним 03/03/18 Суб 23:10:14  386083
>>386075
Хм, кажется я обнаружил потенциальный источник энергии для человечества. Ну-ка:
Ракетоноситель-ракетоноситель.
Аноним 03/03/18 Суб 23:11:06  386084
14194161817540.jpg (391Кб, 1301x950)
>>386081
Вот же.
Аноним 03/03/18 Суб 23:11:14  386085
>>386081
>пик
Пока не пригляделся, думал, что какое-то индусское или буддистское божество.
Аноним 03/03/18 Суб 23:13:00  386086
yoba rocket 2.webm (4084Кб, 640x360, 00:00:16)
>>386083
Прекращай.
Тебя зарепортить что ли?
Аноним 03/03/18 Суб 23:13:41  386087
>>386083
Просто удвоение слишком пошло.
Надо так: Ракета-носитель ракетоносителя (или трёхступенчатая ракета, лол).
Аноним 03/03/18 Суб 23:14:02  386089
>>386086
Всё, больше не буду.
Аноним 03/03/18 Суб 23:16:14  386090
А это, ракетаноситель шаттла?
Аноним 03/03/18 Суб 23:19:28  386092
>>386090
Бегите из треда, сейчас тут ебанёт!
Аноним 03/03/18 Суб 23:19:42  386093
>>385987
Вот попасть по планете как раз сложнее, чем по звезде. Для такого в их машинах смерти должен быть какой-то механизм ускорения до околосвета лишь на расстоянии в 1 световой год до звезды. То есть они пролетают самое долгое расстояние на медляке по навигации, потом просыпаются, чекают, компьютер находит планету и уже тогда дает команду на ублюдочное ускорение. но ок, это все сайфач, больше буду трогать эту тему.
Аноним 03/03/18 Суб 23:24:58  386096
>>386076
В обозримом будущем - нет. Откуда очень сложно взлететь.
Аноним 03/03/18 Суб 23:28:17  386097
>>386050
> нашу каноничную
ТЫ про пикрил? Почему она канонична?
Аноним 03/03/18 Суб 23:33:17  386099
>>386097
Потому, что аниме и была раньше.
Ненавижу этот анимешный мем.
Аноним 04/03/18 Вск 00:21:12  386111
Анон как тебе такая теория?
1 Большой взрыв явление местное
2 после чего материя разлетается,
3 собирается в различные тела,
4 тела собираются в галактики,
5 галактики собираются в суперкластеры как Ланиакея и Пересеи-Рыб
6 в центре масс появляется Великий аттрактор - по сути Сверх ЧД
7 Великий аттрактор пожирает почти все галактики суперкластера
8 Превышение неизвестного нам предела давления
@
1 Большой Взрыв N+1


Теория построена на новой карте суперкластеров
Аноним 04/03/18 Вск 00:26:11  386113
Атмосфера Титана будет сильно мешать взлету оттуда?
Аноним 04/03/18 Вск 00:55:40  386120
>>386111
Пойдет, если ты пятиклассник.

В реальности материя нихуя никуда не собирается, а продолжает удаляться друг от друга с расширением Вселенной. Сверхскопления галактик — не какие-то гравитационно связанные структуры, а просто сгустки плотности, галактики и скопления галактик в них удаляются друг от друга.

Великий аттрактор — просто большое сверхскопление, ничего особенного в нем нет, и Млечный путь, кстати, тоже от него удаляется, как и большинство галактик в его окрестностях, просто помедленнее, чем по закону Хаббла, потому что масса аттрактора немного притормаживает разлет.
Аноним 04/03/18 Вск 01:24:47  386123
>>386120
то есть Земля с каждой секундой становится больше?
если нет, то у тебя устаревшая инфа.
Аноним 04/03/18 Вск 01:25:19  386124
>>386120
Карта Ланиакеи с тобой не согласна но ты можешь бомбить дальше
Аноним 04/03/18 Вск 01:28:25  386125
>и скопления галактик в них удаляются друг от друга.
http://russkievesti.ru/assets/images/resources/6678/640x480/1-0.2016-03-208586757674758866.jpg
как там в двадцатом веке? барышни еще работают на Кюхне?
Аноним 04/03/18 Вск 01:58:05  386128
s7 копия.png (25Кб, 1069x378)
>>386043
Аноним 04/03/18 Вск 01:59:33  386129
>>386051
У Англии подлодки с модифицированными американскими ракетами Трайдент.

Своих нет.
Аноним 04/03/18 Вск 02:13:54  386132
14693138363060.jpg (31Кб, 464x320)
Посоны я как бы знаю что раздел немного не торт, но может ли кто-то поезднить за путинскую стратегическую ракету с ядерной энергоустановкой, что это, как и зачем?
Аноним 04/03/18 Вск 02:17:01  386134
>>386132
Ее никто не видел, существует только в презентации пыни, пояснять просто нечего.
Аноним 04/03/18 Вск 02:26:48  386137
>>386123
>>386124
>>386125
Перепись идиотиков пейсача. В головешках собачка накакала, но мнение имею.
Разница в скорости отклонения от хаббловского расширения - 20% от необходимой для привязки к крупномасштабной структуре. Сама структура гравитационно не связана . Сверхскопление лишь видимость, и по мере эволюции Вселенной Ланиакея растворится.
В следующий раз, прежде чем кукарекать что-либо, поинтересуйтесь вопросом, как минимум. Говноеды с магическим мышлением.
Аноним 04/03/18 Вск 02:29:21  386140
>>386132
сначала это
https://www.youtube.com/watch?v=bME29-_35Bw
потом это
https://www.youtube.com/watch?v=U-T-dPsu9U4
Аноним 04/03/18 Вск 03:22:14  386152
Какие силы вызывают такие отклонения в оси вращения планет в солнечной системе?

Почему ось вращения Земли не перпендикулярна плоскости эклиптикикажется так называется плоскость вращения Земли вокруг Солнца? Почему у Урана она завалилась на почти 90 градусов? Сразу оговорюсь, мои знания по астрономии ограничиваются уроками в 1-3 классах по окружающему миру, поэтому попрошу объяснить доступно.

Почему эти силы не вносят такие же значительные коррективы в плоскость вращения планет вокруг Солнца?
Аноним 04/03/18 Вск 10:45:33  386191
>>386152
Предположительно, результаты столкновений с другими телами.
Аноним 04/03/18 Вск 11:33:11  386194
>>386140
Глянул второе видео на пару секунд.
>маневрирование снижает точность попадания
>ПРО США защищает только авиабазы с ядерными ракетами

Там и вся остальная аналитика на том же уровне или всё же стоит потратить время?
Аноним 04/03/18 Вск 11:33:21  386195
вопрос от тупого, допустим надо закинуть на луну 500 кг груза . что для этого нужно?
Аноним 04/03/18 Вск 11:34:20  386196
>>386195
Космический корабль, который может взять на борт полтонны, буксир и два старта сверхтяжелой ракеты.
Аноним 04/03/18 Вск 11:44:30  386198
это ясно, а именно из существующего пусть немного модифицированного под это дело, реально нет?
Аноним 04/03/18 Вск 11:47:24  386200
>>386198
Если тебе не принципиально, то можно хоть щас протоном пульнуть прямой наводкой. Твои полтонны оттормозятся о реголит. А если хочется без литобрейкинга, то нужен космический корабль. Их пока нет (универсальных на грузы). Если твои 500кг и есть космический корабль с топливом и прочим, то все гораздо проще. Если ты просто мамку решил на пенсию отправить, то нужно городить вокруг нее корабль с посадочным движком, СЖО, батареями и прочим. А он и весить будет помимо 500кг. Тут коварный Циолковский и просит предъявить за счет дельты.
Аноним 04/03/18 Вск 11:50:49  386201
>>386196
А как Союз свои Луноходы одной ракетой и без литобрейкинга запускал?..
Аноним 04/03/18 Вск 11:52:29  386202
image.png (183Кб, 520x326)
image.png (684Кб, 925x653)
>>386201
У них был посадочный модуль.
Аноним 04/03/18 Вск 11:53:08  386203
>>386194
Что не так? Он единственные ошибки допустил, когда предположил что у США есть ПВО которая способна защитить их от крылатых ракет. Лол, у них нет ПВО, есть определенные авиабазы, откуда могли бы взлетать самолеты на перехват ракет, но радарного покрытия всех границ США у них нет. Бояршинов по сути опытней многих тут будет, дохуя пожил, повидал, наслушался.
Аноним 04/03/18 Вск 11:55:41  386204
>>386194
маневрирование реально снижает точность попадания как я читал. Там инерциональная система навигации например на крылатых ракетах, которая со временем накапливает ошибку отклонение. Континентальная ПРО США по твоему вокруг крупных городов стоит? Шахтные хуевины в северных штатах защищает.
Аноним 04/03/18 Вск 12:03:39  386205
>>386202
Луноход-1 весил 700 кг. Плюс посадочный модуль.
И всё это великолепие с АМС в одно рыло доставил Протон.

>>386204
Ну не знаю точно, канеш, но маневрирование, по идее, должно повышать точность попадания. Но это не точно.
А ПРО США, тащемт, в том виде, в котором она сейчас продвигается, рассчитана на сбитие вообще всего. Это в брежневские времена, может, только вокруг ракетных шахт ПРО стояло, ну дак аналогичное есть и в СССР/РФ (правда, прикрывает не шахты, а Москву с Питером). Кипеш стоит именно из-за развёртывания полноценного прикрытия всей, нахуй, страны — что, в перспективе, позволит начинать войну с кем угодно, не опасаясь возможности стать радиоактивным пеплом.
Аноним 04/03/18 Вск 12:19:18  386208
>>386111
Ну и что ты тут построил? Большое сжатие + циклическая модель? Нихуя ты теоретик! Лего не пробовал собирать на скорость? Может ты спящий чемпион в этом деле, м?
Аноним 04/03/18 Вск 12:46:21  386211
>>386205
>из-за развёртывания полноценного прикрытия всей, нахуй, страны
Ну давай запруфай, лол. Где на территории США еще кроме Аляски и пары очень северных штатов стоят инсталяции ПРО в виде ракет-перехватчиков и радаров? И планы давай найди хотя бы. Я подожду.
Аноним 04/03/18 Вск 12:52:35  386213
>>386205
А ещё я слышал что про не спасает на 100%. Т.к. ракета летит с космической скоростью, а про нет. Ввиду этого берут количеством. Это так? Хотя это перетекает немного в другую тему. Достаточно нестабильную, давайте этим вопросом и ограничиваемся.
Аноним 04/03/18 Вск 13:00:39  386214
>>386213
Тред в бамплимите. ПРО имеет процент успешных попаданий по мишеням. Но она не может защитить от массированной атаки, когда две страны уничтожают друг друга всем арсеналом. Там уже такие тактики появляются как взрыв боеголовок в верхних слоях атмосферы, что значительно ухудшает работу любых радаров, а за этим уже летят более тяжелые боевые блоки, которые несут несколько боеголовок и ложные цели. У ракеты мало времени чтобы отличить боеголовку от ложной цели и если она ошибется(а сигнатуры ложных целей созданы чтобы быть похожими на боеголовки), то второго шанса не будет. Поэтому ПРО США в её текущем облике способна отразить очень ограниченную атаку в числе 1-15 ракет старого поколения, на которых не было разделяющихся боевых блоков и множественных ложных мишеней вдовесок.
Но у власти и силовиков еще есть паранойя, что США готовят глобальный удар по нашим ракетам, так чтобы разом при помощи какого-то налета крылатых ракет, беспилотников, НЛО разом уничтожить все 90% наших носителей ракет, обезглавить управление стратегическими силами и вот тогда у РФ осталось бы 10% носителей, которые бы мы смогли в ответочку запустить по США и вот против этих 10% оставшихся ракет ПРО США смогла бы что-то сделать в перспективе лет так эдак через 20...
Аноним 04/03/18 Вск 13:04:02  386215
то есть в мышлении наших силовиков США будут улучшать ПРО в будущем и может в будущем для них будет приемлемыми определенные потери типа "ок, мы пропустим 20 ракет нашей ПРО, но все остальные перехватим и цена, которую мы заплатим за уничтожение РФ в долгосрочной перспективе будет того стоить".

И наших силовиков сложно винить, когда в США есть вот такие люди. Почитай вот
http://thehill.com/homenews/senate/376548-lindsey-graham-war-with-north-korea-would-be-worth-it-in-the-long-run
Аноним 04/03/18 Вск 13:41:24  386227
>>386111
>теория
Иди загугли, что такое теория, пятиклассник
Аноним 04/03/18 Вск 13:44:03  386230
>>386227
ОТОдауна порвало
Аноним 04/03/18 Вск 13:45:24  386231
>>386208
причем тут сжатие олигофренушка
если есть ЧД галактического масштаба почему бы не быть ЧД кластерного и суперкластерного масштаба? боги ОТО не позволяют?
Аноним 04/03/18 Вск 13:46:46  386233
>>386137
то есть ты машешь теорией набитой костылями против вполне созданных и подтвержденных карт?
двадцатый век штука тяжелая, бросай ею упарываться
Аноним 04/03/18 Вск 13:47:04  386234
>>386205
Ну ты блять конченый. Маневрирование точность ему повышает.

А про ближе к границам России нужно, чтобы сбивать ракеты на активном участке траектории
То есть, когда ракета еще не набрала скорость в несколько км/с и не вышла за пределы атмосферы.

Но один хуй, ракета делает это очень быстро, ибо у нее твр побольше, чем у гражданских.
Все эти про - вообще нихуя не панацея.
Они более-менее надежно работают только против колхозных ракет уровня Северной Кореи.

Аноним 04/03/18 Вск 13:48:04  386235
>>386234
не нужно ничего сбивать нужно ввести эмбарго на поставку лекарств дороже ста долларов
и как рукой, как рукой
Аноним 04/03/18 Вск 13:53:33  386236
>>386235
Ага, и сибирской язвой поросят заражать
И еще специальные сайты для отупления людей делать
Аноним 04/03/18 Вск 14:04:53  386247
>>386236
нахуя?
совку было достаточно чтобы его вождям попридержали лекарство для царственной простаты
Аноним 04/03/18 Вск 14:14:08  386250
>>386247
Че бы блять несешь
Аноним 04/03/18 Вск 14:15:02  386251
Что такое "юла"? Время от времени вижу в тредах.
Аноним 04/03/18 Вск 14:21:21  386256
>>386251
united launch alliance
Аноним 04/03/18 Вск 16:39:14  386297
image.png (275Кб, 750x563)
Народ, посните би/шнику профиты зеркальных и линзовых телескопов.
Аноним 04/03/18 Вск 16:47:32  386301
Добавьте в шапку платину про воздушные шары.
>>384205

>>384206
Аноним 04/03/18 Вск 16:47:56  386302
>>386297
В телескопотред зайди, он рядом на доске валяется.
Аноним 04/03/18 Вск 16:48:33  386303
>>386302
Зашел. Там никого.
Аноним 04/03/18 Вск 16:50:44  386304
>>386303
Попробуй еще раз, но на этот раз задай вопрос, потому, что там телепатов нету.
>>366071 (OP)
https://2ch.hk/spc/res/366071.html
Аноним 04/03/18 Вск 17:19:45  386314
Всё равно тред в бамплимите.

>>386211
> Где на территории США
А не нужно на территории США.
SM-3 на кораблях с "Иджис" в СевЛедОкеане и считай, что территория США покрыта. Сбивает в том числе уже покинувшие пределы атмосферы боеголовки.
Да, пока что она так себе справляется с задачей и что-то уровнем выше колхоз-ракет возьмёт вряд ли. Но работа идёт, и рано или поздно какой-нибудь Block-Stopitsot будет поражать цели уверенно. Лучше этого времени не дожидаться.
Плюс Штаты размещают или готовятся размещать ПРО чуть ли не в каждой граничащей с РФ стране и ещё в десятке других. Даже в Мексике, лол.
>>386234
>Маневрирование точность ему повышает.
Что странного? Маневрирование обязательно должно быть неуправляемым?
Первый абзац: http://encyclopedia.mil.ru/encyclopedia/dictionary/details.htm?id=14053%40morfDictionary
>на активном участке траектории
На активном участке траектории сбить ракету проще всего, но это не значит, что больше её никак не достать. См. SM-3, который даже не по ракете лупит, а прямо по боеголовкам.
>Все эти про - вообще нихуя не панацея.
Ну это пока.

Такшта всё правильно РФ делает. При текущем развитии ситуации на руках у РФ должны быть способы доставки ядрён-батона потенциальным друзьям со всех мыслимых направлений, кроме, пожалуй, подземного. Ведь договорились же, сука, как люди, ещё при Союзе: вы прикрываете два объекта на своей территории, мы прикрываем два объекта на своей, и все довольны. Но нет же.

Единственное, что непонятно лично мне — что они будут делать с ракетами на АПЛ. Тут разве что спамить океан "Иджисами", я не знаю.


А если возвращаться к исходному >>386132 вопросу, то для начала следует заметить, что ядерной силовой установкой может оказаться что угодно от апокалиптического прямоточника до йоба-микрореактора с вентилятором. Всё, что о нём известно — он существует (впрочем, если по чесноку, этого тоже не известно). Остальное — спекуляции.
Аноним 04/03/18 Вск 17:51:24  386324
>>386250
ты чем то недоволен?
Аноним 04/03/18 Вск 20:27:54  386350
Если орбита некоторой небольшой ЧД пересечёт орбиту солнечной системы, то какие будут последствия? Какова вероятность того, что ЧД образует с Солнцем двойную систему? Какие условия нужны для этого?
Аноним 04/03/18 Вск 20:35:45  386351
>>386233
Каких еще карт, ты даун, что ли? Открой любой справочник галактик и смотри в столбец красного смещения, и там ты внезапно обнаружишь, что все галактики за пределами Местного скопления имеют положительное красное смещение, т.е. удаляются от нас.

Как, по-твоему, могут скопления галактик во что-то собраться в будущем, если они разлетаются друг от друга и скорость разлета со временем растет, а не убывает? Ты думаешь, что они на твою карту посмотрят и подумают «бля, чет хуйня какая-то, нас в какую-то структуру записали, полетели обратно»?

Никакие крупномасштабные структуры уровнем выше скоплений галактик не являются гравитационно связанными и со временем полностью распадутся, а не схлопнутся, верить во что бы то ни было другое после открытия красных смещений и закона Хаббла — быть дауном.
Аноним 04/03/18 Вск 22:20:02  386374
>>386314
Прямоточный двигатель это йоба вроде твердотопливника только вместо горящего топлива ядерная реакция? Или я обьебался?

И зачем на ракете ядерный реактор? Ей же нужно энергообеспечение на протяжении нескольких месяцев, например
Аноним 04/03/18 Вск 22:23:26  386377
>>386374
Да нет никакой ядерной ракеты, узбагойтесь уже.
Аноним 04/03/18 Вск 22:25:22  386379
>>386374
Ядерный прямоточник позволяет летать чисто на воздухе. При движении на огромной скорости воздух залетает в реактор, нагревается и вылетает с еще большей скоростью, создавая тягу. Грубо говоря, у ракеты будет бесконечное топливо.
Аноним 04/03/18 Вск 22:27:04  386381
>>386379
Кто мешает ему нагретому иметь одинаковый импульс вперед и назад нивелируя тягу?
Аноним 04/03/18 Вск 22:34:33  386387
>>386381
Я не понял нихуя, переформулируй вопрос.
Аноним 04/03/18 Вск 22:36:35  386389
>>386387
Воздух попал внутрь и нагрелся.
Нагревшись расширился. Почему он расширяется только назад, а не во все стороны?
Аноним 04/03/18 Вск 22:37:44  386390
>>386389
Это шутка такая?
Аноним 04/03/18 Вск 22:40:57  386392
image.png (47Кб, 1548x683)
>>386390
Ну вот. Ты гришь что от нагрева будет ускоряться (1), но воздух и в обратную сторону расширяется и замедляет входной поток с такой же скоростью с какой ускоряет выходной - скорость должна просто замедляться потому, что неоткуда браться ускорению.
Аноним 04/03/18 Вск 22:42:30  386393
>>386390
Пользуюсь тем, что это тред тупых вопросов:
А правда, анон, можешь на пальцах для особо тупых пояснить, как так получается? Я в курсе, что прямоточные реактивные двигатели работают, но почему они работают - не понимаю. Ведь и правда, нагрев же должен воздух во все стороны гнать, а не только к соплу?
Аноним 04/03/18 Вск 22:47:04  386396
>>386389
>>386392
>>386393

Потому что спереди его толкает встречный поток в несколько махов скоростью, а сзади норм, туда надо лететь.
Аноним 04/03/18 Вск 22:48:27  386397
>>386396
Толкает. Но внутри сопла нагретый толкает встречный поток назад немножко и немножко вперед, откуда эффект тяги-то?
Аноним 04/03/18 Вск 22:56:34  386399
>>386397
Нихуя он не толкает вперед, зачем ему вперед идти если там махов пять, а сзади ничего?
Аноним 04/03/18 Вск 22:57:37  386400
>>386399
Потому, что расширяется во все стороны одинаково, ты че.
Аноним 04/03/18 Вск 22:58:33  386402
>>386399
Легче ему будет объяснить на примере ДВС, где вместо клапанов охуевшее давление со стороны воздухозаборника.
Аноним 04/03/18 Вск 23:01:32  386403
>>386399
Ну допустим, пять махов спереди поначалу сдует нагретый воздух в правильном направлении. Но импульс-то он даст в обе стороны, ёпта, сколько пошло в сторону сопла, столько пошло и против набегающего воздуха. Итоговый импульс нулевой получается.
Тут явно есть какая-то неочевидная тонкость, анон, потому что в чистом виде такое объяснение тянет на мюнхгаузеновский метод самовытаскивания, только почему-то работающий на практике.
Аноним 04/03/18 Вск 23:10:35  386406
>>386400
Ну бля, спереди есть сопротивление, сзади нет. Вот он и вылетает взад.
>>386403
Бля поток же постоянный. С каких хуев он вообще должен вперед идти, если он как шел назад, так и идет, только температура повысилась.
Аноним 04/03/18 Вск 23:11:30  386407
>>386403
>пять махов спереди поначалу
Не поначалу. ПВРД не работает при низкиз скоростях. Любой аппарат с ним сначала разгоняют до скорости при помощи другого движителя, при которой он становится эффективным. Почитай википедию - там достаточно подробно описано.
Аноним 04/03/18 Вск 23:13:46  386408
>>386406
Похуй на сопротивление. См картинку еще раз >>386392
Ты утверждаешь, что твой воздух нагреваясь расширяется только назад, не давя на входящий поток?
Аноним 04/03/18 Вск 23:17:19  386409
>>386408
Да.
Потому что :
Воздух уже движется с огромной скоростью еще до нагрева.
Спереди на него давит поток с огромной скоростью.
Аноним 04/03/18 Вск 23:19:25  386410
>>386406
>Бля поток же постоянный.
Так он очень скоро перестанет быть постоянным, если, кхм, "толкающая сила" от расширения воздуха довит не только в сторону сопла, но и на него самого.
Идти-то он под напором воздуха будет назад, да, но вот если всё буквально так происходит, то движок получается безтяговым, прущим только по инерции.

Можно (для упрощения) представить расширяющийся воздух как Брюса Уиллиса, который упирается руками с двух сторон в одинаковые малолитражки и раздвигает их от себя с одинаковым усилием. Так вот, по принципу прямоточного двигателя получается, что если мы дополнительно пихнём одну из малолитражек в сторону Брюса Уиллиса (создадим набегающий поток), то вместо того, чтобы продолжать с одинаковой силой расталкивать обе малолитражки, он будет усиленно толкать только одну из них, противоположную набегающей. А должен бы продолжать распихивать обе, пока трение не остановит всю конструкцию.
Аноним 04/03/18 Вск 23:19:43  386411
>>386409
Двачую. Пошёл в жопу этот Ньютон с его равенством, что он понимал в своем каменном веке!
Аноним 04/03/18 Вск 23:24:30  386414
>>386410
>наркоманские примеры
Поставь табуретку около стены и встань боком к ней между ней и табуреткой. Упрись внешним боком одной стопы к табуретке, другой - к стенке. Попробуй сесть на шпагат и посмотри, насколько сильно отодвинется стул и стена от твоего начального положения.
Чем я занимаюсь нахуй в этом треде, я ведь даже не физик. Всё, я пошёл читать комиксы, а ты - пошёл нахуй.
Аноним 04/03/18 Вск 23:29:00  386417
>>386414
Кхм. Но ведь я довил с одинаковой силой и на то, и на другое. Стена осталась на месте только за счёт вопиющего превосходства в массе, свою порцию довления она получила, и она равна полученной табуреткой.
Если бы мы втроём висели в воздухе и стена на меня пёрла, она бы после такого номера стала переть чуть медленнее. При расходовании достаточного кол-ва табуреток она в конце концов остановилась бы.
Аноним 04/03/18 Вск 23:35:39  386421
>>386417
>Стена осталась на месте только за счёт вопиющего превосходства в массе, свою порцию довления она получила, и она равна полученной табуреткой.
В ПВРД тоже самое - рабочее тело сгорая оказывает давление на входящий поток, но он как стена, давление создаваемое им - слишком велико и "сдувает" сгорающее топливо. Это как ёбаный самосвал против легковушки, которые несутся друг другу навстречу - после того, как легковушка врежется в самосвал, он будет везти её ещё добрых 100 метров. Ну и конструкция камеры сгорания, конечно, играет роль в снижении этого противодавления в сторону воздухозаборника.
https://ru.wikipedia.org/wiki/%D0%9F%D1%80%D1%8F%D0%BC%D0%BE%D1%82%D0%BE%D1%87%D0%BD%D1%8B%D0%B9_%D0%B2%D0%BE%D0%B7%D0%B4%D1%83%D1%88%D0%BD%D0%BE-%D1%80%D0%B5%D0%B0%D0%BA%D1%82%D0%B8%D0%B2%D0%BD%D1%8B%D0%B9_%D0%B4%D0%B2%D0%B8%D0%B3%D0%B0%D1%82%D0%B5%D0%BB%D1%8C
Почитай, не ленись. Всё очень просто, на самом деле, я даже не знаю, почему здесь не справляется твоя интуиция.
Аноним 04/03/18 Вск 23:45:17  386426
Безымянный.png (23Кб, 2078x824)
Вот смотрите. С земли мы видим точку А. Что мы будем видеть, когда попадем в точку A? И что происходит в точке B?
Аноним 04/03/18 Вск 23:47:14  386428
>>386426
В точке А мы будем видеть все в пределах сферы ограниченной этим расстоянием на картинке, в т.ч. то место где будет земля и что было в точке В 13 гигалет назад.
Аноним 04/03/18 Вск 23:53:07  386433
>>386421
>я даже не знаю, почему здесь не справляется твоя интуиция
Ну вот я вижу, что оно существует, но какого хрена оно существует — ну вот вообще не вдупляю.
Самосвал, набежавший на легковушку, хоть и провезёт её n-ное время, затормозится. И надо учитывать, что поток легковушек нескончаем, а самосвал разгона больше не получает.

Статью на Вики читал, но всё равно не понял, почему тяга идёт только в одну сторону, если должна в обе. Ну сдует набегающий поток расширяющийся газ, но с каждой итерацией набегающий поток должен быть всё слабее, ведь он тоже свою порцию довления получает. А он не слабеет, сук. Чувствую, что подвох в центральном теле и создаваемых им эффектах, но переложить с языка интуиции на что-то вменяемое не могу.

Тут, походу, на пальцах его принцип попросту не объяснить, нужно всерьёз физику курить.
Аноним 04/03/18 Вск 23:58:49  386437
>>386433
Расскажи, если вдуплишь, ты не один щас этим вопросом интересовался.
Аноним 05/03/18 Пнд 00:02:55  386439
>>386433
>Самосвал, набежавший на легковушку, хоть и провезёт её n-ное время, затормозится.
Если аппарат начнёт терять скорость, то так и будет, правда конструкция камеры сгорания при половине скорости звука всё ещё будет эффективно нивелировать противодавление. Если представить себе, что камера представляет собой чуть более технологичное изделие, чем на твоём рисунке здесь >>386392, то на низких скоростях будет так, как ты говоришь.
>что поток легковушек нескончаем
Как и поток самосвалов.
>с каждой итерацией набегающий поток должен быть всё слабее
С чего бы ему быть слабее? Сотни километров девственной атмосферы, спокойно ожидающей, пока об неё не врежется воздухозаборник, в котором она создаст ебическое давление.
>А он не слабеет, сук
Аппарат ускоряется, давление в воздухозаборнике увеличивается, а давление в сопле все ещё больше давления вне сопла. Когда скорость аппарата выравнивается со скоростью истечения струи, насколько я понял, разница в давлении между "внутри сопла" и "вне сопла" становится достаточно малой и эффективность ПВРД падает, проходя, сообственно, свой пик эффективности.
Аноним 05/03/18 Пнд 00:03:30  386440
>>386433
>И надо учитывать, что поток легковушек нескончаем, а самосвал разгона больше не получает
Надо учитывать что самосвал получает разгон от леговушки, так как тяга из сопла увеличивает скорость двигателя относительно воздуха.
Аноним 05/03/18 Пнд 14:21:55  386514
>>386351
> ррря вреети ваши карты не карты ваши данные не данные!!
ясненько
Аноним 05/03/18 Пнд 14:31:11  386515
Стикер (191Кб, 500x500)
>>386426
Бамп. Что мы будем видеть в точке В, если прямо сейчас телепортируемся туда? Когда будем смотреть дальше по красной линии.
Аноним 05/03/18 Пнд 14:36:58  386519
>>386515
В точке А и Б будет примерно такой же вид как с Земли.
Аноним 05/03/18 Пнд 14:49:33  386522
>>386519
Но в точке А, я же на границе вселенной буду, почему такой же?
Аноним 05/03/18 Пнд 14:58:42  386527
>>386522
Ты и щас на границе обозрения всех точек, которые находятся на границе твоего обозрения.
Аноним 05/03/18 Пнд 17:02:43  386589
>>385971
Смешали все в одну кучу, нельзя так.

Предельно упрощённое объяснение числа Маха
Для понимания числа Маха неспециалистами очень упрощённо можно сказать, что численное выражение числа Маха зависит, прежде всего, от высоты полёта (чем больше высота, тем ниже скорость звука и выше число Маха). Число Маха — это истинная скорость в потоке (то есть скорость, с которой воздух обтекает, например, самолёт), делённая на скорость звука в конкретной среде, поэтому зависимость является обратно пропорциональной. У земли скорость, соответствующая 1 Маху, будет равна приблизительно 340 м/с (скорость, с использованием которой люди оценивают расстояние до приближающейся грозы, измеряя время от вспышки молнии до дошедших раскатов грома) или 1224 км/ч. На высоте 11 км из-за падения температуры скорость звука ниже — около 295 м/с или 1062 км/ч.

Такое объяснение не может использоваться для каких бы то ни было математических расчётов скорости или иных математических операций по аэродинамике.
Аноним 05/03/18 Пнд 17:39:13  386592
EmDrivebuiltbyE[...].jpg (37Кб, 437x408)
А что там слышно про ЕМ-драйв? Есть какие-то новости? Когда мы уже начнем бороздить космос без химических коптилок?
Аноним 05/03/18 Пнд 17:41:04  386593
>>386592
Секретная нагрузка для флакона под кодовым именем Зума была не президентом бантустана и не игрой про пепе, а волшебным ведром.
Ты знаешь, что с ней стало.
Аноним 05/03/18 Пнд 17:49:37  386594
>>386593
Ой, ну я серьезно спрашиваю. Твоя версия охуенна, конечно, но слишком хороша, чтобы быть правдой, как и мнение, что ем-драйв на Х-37 испытывали.
Аноним 05/03/18 Пнд 17:52:04  386597
>>386594
Нет, всерьез теории нет, китаезы чето вякали, но никто ничего не запускал. Это надо не один мильон всадить на проект который по мнению ученых которые занимаются подобными вещами скорей всего не выстрелит нифига.
Аноним 05/03/18 Пнд 18:03:25  386603
>>386597
Жаль, все таки на задворках сознания хочется надеяться, что это именно тот случай, когда эмпирические наблюдения опережают теоретический базис. Интересно было бы, если кто-то все-таки кубсат запустит с этой штуковиной.
Аноним 05/03/18 Пнд 18:05:25  386604
>>386603
У меня небольшая паранойя, что эта штука работает даже лучше, чем было сказано, потому независимым проверяльщикам просто не дают запустить кубсат и военные уже вовсю меряются письками и создают супероружие на базе ведра.
Но как обычно реальность гораздо банальней.
Аноним 05/03/18 Пнд 18:28:09  386608
>>386514
Ты ебучий дурачок, увидел картинку, но не смог ее интерпретировать. Хотяв общем-то взять туже статью из которой та самая презенташка, то там человеческим языком для даунов все разжевано.
Аноним 05/03/18 Пнд 18:29:49  386610
>>386608
А что заставлеят их разлетаться? Какая сила? Ведь без силы они бы притягивались ведь притяжение действует бесконечно.
Аноним 05/03/18 Пнд 18:39:30  386612
>>386610
https://ru.wikipedia.org/wiki/Расширение_Вселенной
Аноним 05/03/18 Пнд 18:40:16  386613
>>386610
https://ru.wikipedia.org/wiki/Тёмная_энергия
Аноним 05/03/18 Пнд 21:07:40  386665
image.png (16Кб, 540x584)
>>386437
Я, кажется, понел, в чём была моя (наша) ошибка при рассуждениях.
Я ввёл избыточное упрощение, которое похерило суть ракеты. При разговорах о прямоточнике я воспринимал его как тупо трубу, где-то в середине которой поджигается топливо и мистическим образом расширяется только в одну сторону. А фишечка, походу, всё же как раз была в центральном теле (конус такой в воздухозаборнике) и в том, что я его не принимал в расчёт ради упрощения (собсна, как на картинках того >>386392 анона; труба буквальная). Зря.

Вон слева боевая картиночка, поясняющая суть™ так, как я её "понял", а заодно ход моих рассуждений.
Под а) у нас тупо банка, выхлоп из которой в принципе может идти лишь в одну сторону. Тут всё предельно просто и интуитивно понятно, такой себе детсад-левел ракетный двигатель.
Под б) вводим усложнение в виде дырки отверстия дырки в передней части банки. Безо всяких физических выкладок: очевидно, что часть мощности пойдёт в эту дырку и снизит итоговую эффективность реактивной банки, но не фатально; если не перебарщивать с размером дырки, банка будет продолжать лететь в нужном направлении, пусть и медленнее варианта а).
Под в) у нас, собственно, быдло-левел прямоточник. Неожиданно, он концептуально сильно схож с банкой типа б): основное сопло сзади, мелкие отверстия спереди. Посередине воздухозаборника увесистое центральное тело, играющее заодно роль передней стенки банки. Собственно, в нём и крался подвох: я ради упрощения не учитывал наличие центрального тела, чему, справедливости ради, весьма способствовало словесное описание прямоточника как "трубы".
Правда, полученный таким путём размышлений вывод несколько отличается от попыток что-то мне втолковать от анонов выше: импульс от расширяющегося газа действительно идёт в обе стороны, а не только в одну, просто со стороны воздухозаборника он в значительной степени отражается центральным телом; таким образом, итоговый импульс назад значительно перевешивает импульс вперёд, проблема решена. Продолжая путь рассуждений, получается, что забор воздуха спереди действительно снижает общую эффективность двигателя, но тут уже всё упирается в необходимость доставить в камеру сгорания воздух, а при глухой стенке вместо воздухозаборника это проблематично, см. "ракетный двигатель".
Упреждая могущий возникнуть вопрос о причинах неработоспособности прямоточника на низких скоростях, можно предположить, что если набегающий со стороны воздухозаборника поток воздуха недостаточно силён для того, чтобы сдувать продукты сгорания в сторону сопла, последние будут вываливать через воздухозаборник вместо воздуха сами и движок не получит кислорода для своей работы. Именно поэтому ему нужна скорость.

Знающий анон, рассуди, пожалуйста, да укажи на ошибки; только помни, что это тупых вопросов тред и перед тобой отпетый гуманитарий с формулофобией.
Аноним 05/03/18 Пнд 21:11:04  386666
>>386665
>как на картинках того >>386392 анона
Яэто и есть >>386437
Хорошо расписал, стало яснее. Или может я трезвый просто.
Аноним 05/03/18 Пнд 21:31:31  386673
>>386522
Не на границе вселенной, а на горизонте. За горизонтом ничего не видно, но когда идешь к тому месту где горизонт - он отодвигается.
Аноним 05/03/18 Пнд 23:01:17  386718
Почему когда Луна уменьшается полумесяц наподобие буквы С, когда увеличивается - наоборот?
Аноним 05/03/18 Пнд 23:14:35  386726
тред пора переименовать в НАУЧНЫЕ КОСТЫЛИ НАШЕГО ВРЕМЕНИ
Аноним 05/03/18 Пнд 23:16:06  386727
>>386612
если все расширяются из одной точки сложно объяснить нередкое явление столкновения галактик -ведь из траектории строго радиальны
тяжело жить в мире из костылей))
Аноним 05/03/18 Пнд 23:17:16  386728
>>386608
там сказано что по собранным данным было выяснено что движение галактик разнонаправленное к чему были нарисованы синие и красные стрелки
но средневекового варвара было уже не остановить)
Аноним 06/03/18 Втр 00:14:35  386749
>>386728
>>386727
Ты слишком тупой даже для этого треда. Но не это грустно, печально, что ты абсолютно убежден в своих заблуждениях.
Не вдуплять понятия пекулярных скоростей, и называть кого-то средневековым варваром в пейсаче - это надо быть ебанутым наглухо.
Аноним 06/03/18 Втр 00:17:46  386752
омега.jpg (54Кб, 604x340)
Что произойдёт, если в ограниченном объёме какой-то материал разогреть до планковской температуры?
Аноним 06/03/18 Втр 00:19:06  386754
>>386752
Фонить будет ояебу.
Аноним 06/03/18 Втр 00:21:45  386758
>>386749
>пекулярных скоростей
для постулирования одних костылей ты размахиваешь другими костылями
замшелый академизм как есть
Аноним 06/03/18 Втр 00:23:03  386760
>>386754
Если этот объём будет скажем куб см, расплавится ли кора, хотя бы частично?
Аноним 06/03/18 Втр 00:28:07  386761
Не нравится - и[...].webm (1776Кб, 240x180, 00:01:14)
>>386758
Аноним 06/03/18 Втр 00:31:05  386763
>>386760
Если кубический сантиметр говна растворить в озере 10х10 км, будет ли озеро отчетливо вонять говном?
Аноним 06/03/18 Втр 00:32:39  386764
>>386763
если озеро толщиной в пару микрометров или говно концентрированней обычного в миллион раз - то будет
Аноним 06/03/18 Втр 00:38:39  386765
>>386764
Данное говно же концентрированее обычного в 10 в 32 раза. Вопрос лишь в том сколько энергии не улетит в космос, а преобразуется в тепло.
Аноним 06/03/18 Втр 00:40:45  386767
>>386765
>Данное говно же концентрированее обычного в 10 в 32 раза
>>386763
>кубический сантиметр
Да оно мощнее солнца, друже.
Коллапсирует и красивое огненное шоу на ближайшуюю тысячу парсек устроит.
Аноним 06/03/18 Втр 01:05:11  386773
>>386767
>Да оно мощнее солнца, друже.
Ой ли?
>Коллапсирует
C чего вдруг?
>красивое огненное шоу на ближайшуюю тысячу парсек устроит.
Охуительные истории.
ПЕРЕКОТ Аноним 06/03/18 Втр 01:59:17  386778
Перекат.

>>386777 (OP)
https://2ch.hk/spc/res/386777.html
Аноним # OP  06/03/18 Втр 02:09:13  386788
>до переката
>полтора поста в час
>после переката
>ушат говна по посту в минуту
Вот поэтому я не хотел тред перекатывать.
А на самом деле мне было лень.
Я и не перекатывал, другой анон взялся.
Аноним 06/03/18 Втр 02:12:16  386789

Вы прослушали исповедь вниманиеблядка.
Аноним 06/03/18 Втр 02:13:41  386790
>>386789
Ты про че щас?
Аноним 18/03/18 Вск 01:58:23  389711
>>384479
в чем отличия, между зелеными и красными?
Аноним 26/05/18 Суб 22:30:49  403098
blob (97Кб, 246x205)
Реально ли было на пиндосовском шатле совершить облет или посадку на Луну, если бы в качестве полезной нагрузки взять на борт дополнительное топливо?


Топ тредов
Избранное